Vous êtes sur la page 1sur 123

NMAT REPEATED

QUESTIONS
Probability of NMAT Questions getting repeated in the the various NMAT
Windows is very high. The Document contains all the questions which
have been repeated in the NMAT Windows- 1 to 4 , 2016-2017

Admito

NMAT REPEATED QUESTIONS

MBA College Selection Made Easier

NMAT REPEATED QUESTIONS

VOCABULARY WORDS
Window 1
1. Word : Obtrude, Meaning: to agree, to adapt oneself Synonym: confirm to, adhere
to, obey with, follow. iAntonym: oppose, revolt against, mutiny against, scoff at, wink
at, refuse, challenge
2. Word: Puissant. Meaning: powerful. Synonym: , mighty, having authority, potent.
Antonym: (Impotent, feeble, flimsy, unskilled, unqualified, unfit, powerless
3. Word: Comply nt, urbane. Synonym: clumsy, awkward, rustic
4. Word: Differential. Meaning:varying according to circumstances. Synonyms:
discriminator Antonyms: equal, equitable, fair, just, impartial,neutral, unprejudiced
5. Word: Deferential. Synonym: respectful, dutiful. Antonym: disrespectful, undutiful
6. Word: Gauche. Meaning: socially awkward, ill mannered. (opposite :elegae - to
impose something on someone. Synonyms : butt in, interlope, intermeddle, intrude,
meddle, mess, muck (about or around), nose, interfere, poke, pry. Antonyms : avoid,
eschew, shun; disregard, ignore, neglect, overlook
7. Word: Obscure. Meaning: having an often intentionally veiled or uncertain meaning,
difficult to understand. Synonym: dark, dull, not clear,hidden, mysterious,
ambiguous, equivocal, murky, mystic.) Antonym: clear, obvious, unambiguous,
unequivocal,elucidate)
8. Libertine - Playboy ,lecherous, freedman; characterized by free indulgence in
sensual pleasures (Antonym: pure, uncorrupt)
9. Mendacity - lack of honesty, untruthfulness. Synonym: fable, falsity, whopper
10. Denial - TO REFUSE Enervate - weaken-, to sap energy from. Synonym- debilitate.
Antonym: beef up, fortify, strengthen, buttress
11. Obfuscate make unclear, bewilder, to make obscure. Synonym: confuse, perplex,
befog, blur, fog, muddy). Antonym: clarify, clear, illuminate
12. Alacrity eagerness, zeal, avidity, liveliness, willingness. (Antonym- Apathy)
13. Insouciant-unconcerned,
nonchalant,
cal.,
Opposite is Anxious/Concerned/ cautious/heedful/wary

lackadaisical.

NMAT REPEATED QUESTIONS

14. Truculence / Truculent : eager or quick to argue- (syn- aggressive, antagonistic,


assaultiveness, contentiousness, defiance, cruel and ferocious, barbarity, to atrocity,
cruelty) (Antonym: benignity, compassion, sympathy)
15. Prevaricate -speak or act in an evasive way; escaping or avoiding way, lie (Syn:
fabricate, fib) (Ant: swear, testify, authenticate, confirm, validate, verify)
16. Intractable - hard to control or deal at (someone) is not guilty of wrongdoing. (Syn:
defiant, disobedient, noncompliant, rebel, wayward, wilful) (Ant: obedient,
submissive, compliant, docile)
17. Vagaries - unpredictable action (Syn: fancy, caprice, freak, whim, whimsy)
(antonym: actuality, reality, verity)
18. Culpable - deserving blame. syn:blamable,blameworthy,reprehensible,reproachable)
(ant:flawless,guiltless)
19. Facetious - treating serious issues with deliberately inappropriate humour , antonym
- serious
20. 22. Nihilism - the rejection of all religious and moral principles, often in the I belief
that life is meaningless Philosophy- the belief that nothing in the world has a real
existence - antonym-belief, faith
23. Skew- to change so that it is not true or accurate, usually in order to favour a particular
group
24. Hierophant- priest
25- Puerile- immature, juvenile
26- Plebeian- a commoner, worker class, hoi polloi (
)
, antonym - noble(Synonym asked- Answer was common people)
27- Altruistic-

showing selfless concern for welfare of other, unselfish

28- Calumniate -Utter malicious words, make false and defamatory statements about
others.eg -kejriwal(u will never forget the meaning hereafter)
(Antonym asked- Extol)

NMAT REPEATED QUESTIONS

29 - Stumblebum -inept person, clumsy, done without skill or experience. Antonym-adept,


skilled
30- Verbiage: excessive and meaningless usage of words (in a speech or language).Ant.:
succinct, laconic.
31- Soliloquy : an act of speaking one's thoughts aloud when by oneself
or regardless of any hearers, especially by a character in a play., a speech
32- Colloquial:-

informal, not formal, argot

33- Scorn - Lack of respect accompanied by a feeling of strong dislike


34. Disdain -contempt, scorn,disrespect (Antonym : admiration, respect)
35. Extol - To praise enthusiastically (Synonym: Exalt, Praise,adulation)
36.Trenchant - Incisive or vigorous in style or expression,i7, sharp (Antonym : Vague)
37.Imprecation - a spoken curse ,a magical word or phrase uttered with the intention of
bringing about evil or destruction (syn- Malediction)
38. Sycophant -servile(a person who does excessive flattery, creep i.e he does sycophancy),
adulate
39. Adulate -Excessive Flattery( Salman khan have adulatory fans)(For eg-The celebrity was
overwhelmed,by the adulations he received from his fans)
40. Skeptic -doubtful
41. Insoile - Reserved(Q. Is this a word?,not able to find anything in google) its maybe
insolite meaning (unusual french word)
42. Impecunious:- having little or no money, penniless
43. Exculpate - show or declare that (someone) is not guilty of wrongdoing. (Antonym was
asked)
(Ant.)- accuse, arraign, charge, impeach, indict; convict, criminate, incriminate
44. Arbiter - a person who settles disputes / impose.
45. Chaos :complete disorder and confusion.

NMAT REPEATED QUESTIONS

46. Procrastination - the action of delaying or postponing something.


Antonym Anticipication
Consummate: Antonym: unskillful, amateur, unprofessional
Synonym :perfect, complete, Quintessential
syn came in 27 oct and 28th and repeated 2nd nov repeated on Nov 5 first slot(syn)
2. Saturnine: Antonym: cheerful, jovial
Synonym : unhappy , very serious, lugubrious, sombre
came in 27th and 28th , Synonym came on 28th and repeated 2nd nov [extra- lugubrious
and sombre both mean dull], Repeated 2nd nov, synonyms were asked)
3. Expostulate: Synonym: Argue, disagree, remonstrate, express strong disapproval or
disagreement
Antonym: approve, sanction, accept,
Repeated 2nd nov, synonyms were asked, options had tough words, and it was not easy to
select the right one.
came in 27th and 28th
4.Insouciant(Synonym)- unworried,casual.easy going,nonchalant, equanimous.
(Antonym: caring, sensitive,concerned)
The word insouciant repeated in 25 oct morning slot antonym was asked options i don't
remember
This word got repeated today(28th) antonym
5.Disdain- synonym: unworthy, scorn, disparagement
antonyms: Admiration, Respect, Compliment, Praise.
6.Exculpate -meaning: free from guilt, flawless
antonym : faulty
synonym : exonerate, emancipate,vindicate,absolve [exonerate- absolve (someone) from
blame for a fault or wrongdoing.] Repeated 28 th
7.Obtrude-synonym:interlope,intrude,mess,infringe, impose, shove, force push
antonyms:avoid, eschew,ignore, neglect,Repeated on 28th
8.Calumny - synonym:defaming,maligning,smearing,vilification/vilifying, slander,
misrepresentation
Antonym: acclaim, accolade, applause, praise, honor,esteem, adoration////. [Repeated
27,(28th oct- synonym) , 29(antonym), 2nd Nov (syn),4th nov -syn-tricky options
9.Whimsical- synonym : not serious,capricious,freakish, impulsive (playful person) repeated
27 oct .Syn or Ant?What was in option? (28th repeated- synonym)
Antonym: down to earth, reasonable, equable (boring, sad) Repeated on 2nd nov

NMAT REPEATED QUESTIONS

10.Astute-(Synonym): Clever, Quick-Witted,perspicacious-28th repeated


(Antonym): Stupid
11.Lackadaisical- (Synonym) : Careless/Lazy , Lacking enthusiasm, insouciant, blase,
offhand repeated 27 oct .Syn or Ant?What was in option? (28th repeated- antonym) ,
Repeated 29 (antonym)
(Antonym): Excited/enthusiastic
12.Exigent-(Synonym) : Pressing or Demanding //Antonym or Synonym asked?
(Antonym) : Unpressured, Easy, facile
13. Abjection - degradation : A LOW OR DOWNCAST STATE, DEGRADATION ,
DEMORALISATION antonym was asked Antonyms : goodness, morality, 28th as well
(antonym)
Repeated on 2nd Nov,5 th Nov Abjection-degradation was the answer.

14. Quagmire - Synonyms: Dilemma,slack, mire, deadlock,impasse, //Antonym or Synonym


asked?
bind, box, catch, corner,hole, spot, sticky-wicket, Antonym:-Agreement/Solution, fix.
15. Obfuscate - obscure : to make something more difficult to understand , becloud, blur,
muddy, confuse,Antonym:-clarify,enlighten, 28th, Repeated on 4th Nov. (Elucidate was in
options) [elucidate-make something clear]
16. Instigate : provoke, abet, ferment, incite, Antonym:Deter,Discourage
17. Imbecile : stupid,feeble minded,backward Antonym:-Intelligent,Smart,Brainy
18. Defalcate : embezzle,steal,misuse,abstract,pilfer Antonym:-Give,receive,compensate
19. Perpetuate : (Antonym was asked ) : synonym : continue, conserve, maintain
Antonym: halt, stop, terminate
Terminate was one of the options, Question repeated today 2nd Nov
20. Proletariat: working class people regarded collectively (often used with reference to
Marxism ),lower class,commoner,peasant,Plebeians. Antonym:- Aristocracy,
Nobility,bourgeoisie.
21.Deference(synonym was asked): respect,reverence,submission,acquiescence,docility.
Antonym:-Fight, Dishonour ,Disobedience. 27th oct repeated (repeated on 4th)

NMAT REPEATED QUESTIONS

22. Insatiable: Synonyms: impossible to


satisfy,greedy,unsatisfied,yearning,gluttonous,voracious,prodigious, ravenous, rapacious
antonyms: satisfied,fulfilled,satiable,full (repeated on 4th)
23. Resurrection: Synonym- rising from the dead, restoration to life, return from the
dead,rebirth,reincarnation,revival "the resurrection of Jesus,Antonym: Death,Expiration
24.Accrue: Synonym : result in / accumulate. Antonym: Spend /decrease/diminish
25.Hackneyed:Synonym:Overused/Overworked. Antonym : Original/Fresh/in its main form
26. Scabrous:Synonym : Having a rough surface,indecent, salaciousAntonym : smooth,clean
27. Fortifying : Synonym:-Build up,protect,strengthen Antonym :- Harm,injure,Hurt,inimical
28. Antediluvian : Synonym :Old Fashioned Antonym : Modern Repeated 27,28Oct Antonym
was asked, modern was in option , Repeated 29 Antonym (Modern), Repeated 2nd Nov,4th
nov
29.
Obdurate :-Synonym : Adamant,Stubborn,Obstinate, RECALCITRANT, Firm.
Antonyms : Yielding, Placid, Plaint, Gentle,Submissive,Susceptible Asked again on 28th
30. Perfidy :, (synonym)-deceitful & untrustworthy (antonym) -faithfulness, loyalty ( 26th oct
morning slot) 27th oct repeated (repeated on 4th. Synonym.)
Question repeated today, Remember inimical and fortifying as opposites, both were asked.
Inimical: Synonym: Hostile; Antonym: Friendly
31.Abstemious (synonym); abstinent, restrained, not eating or drinking too much, self
denial. (Antonym): self indulgent. Synonym or antonym ? Repeated on 4th Nov.
32.Preclude: (syn)- prevent from happening,make impossible,debar (Ant)-allow,make
possible,support
Repeated 27th exam,28th, 4th Nov.
33.Levigated Repeated 28th exam ---means-reduce (a substance ) to fine powder or paste,
Synonyms- Pulverize,grate,reduce,Nov 5(syn)
Antonyms-build,liberate,free
34. Alacrity - eagerness, zeal , avidity, liveliness, willingness (Antonym- Apathy)

NMAT REPEATED QUESTIONS

35. Minacious - threatening, alarming, menacing (repeated on 4th. Synonym. Options had
menacing)
36. Levitate- rise into the air,be suspended,hover, soar,up rise Antonym - land, descend
37.perpetual - same as perpetuate.(ever lasting , continue ) correct.
38. Deferential- yes this word was in Window 1 too!
Synonym : Courteous,Polite,showing Respect
Antonym: Rude,Impolite
39. assuage-Synonym-satisfy, ease (an appetite or desire) Antonym-intensify- was asked on
2nd Nov
NOV 3 SAME WORDS FROM DOC:
INSOUCIANT (antonym) - Heedful
INIMICAL (syn)
DEFALCATE (syn)
PERPETUAL (ant)
OBDURATE (ant)
NMAT WORD ON 6th: ASPERSE

NMAT Window 3
ENERVATE (was in w-1)- weaken-, dauntless to sap energy from. (Synonym- debilitate,
Antonym: arbeef up, fortify, strengthen, buttress
Puissant (adj)- powerful, mighty, having authority, potent. (Antonym was asked) Impotent,
feeble,
Flimsy
2 If Veracious-brave truthful true, accurate, veritable, correct, errorless, unerring, exact,
precise, factual, literal, realistic, authentic, faithful, close, strict, just, unelaborated,
unvarnished;
If Voracious-having a great a appetite, unable to satisfy

NMAT REPEATED QUESTIONS

synon insatiable, unquenchable, unappeasable, prodigious, uncontrolable ,omnivorous,


yms: compulsive, glutton
3 Stumblebum -inept person,clumsy, done without skill or experience. Antonym-adept,
skilled
4.Proletariat -working class people regarded collectively (often used with reference to
Marxism ),lower
class,commoner, peasant, Plebeians. Antonym:- Aristocracy, Nobility,bourgeoisie
5.Circumspect-synonym-cautious, wary, careful, chary, guarded, on one's guard; 12 th syn
antonym-unguarded, incautious (12th,11.30- antonym with option
incautious)
6. Opprobrium : harsh criticism or censure. Synonyms: abuse, condemnation, criticism,
defamation, slander, obloquy, bad press. Antonyms: praise (12th,11.30-synonym was asked)
7. Ignominy : public shame or disgrace. Synonyms: shame, humiliation, embarrassment,
disgrace, dishonor, discredit, degradation, scandal. Antonyms: honor
7. Cantankerous : Synonyms: bad-tempered, grumpy, fractious, argumentative,
uncooperative, to oppose Antonyms: good natured, affabl0e 12 th syn
8. Fractious: Synonyms : Obstreperous (noisy , difficult to
control),awkward,irritable,wild,unmanaged, problem maker Antonym Contented,agreeable,nice++
9. Surreptitious - syn - secretive, clandestine, Antonym - Blatant (
10.Labyrinthine(Antonym)-simplicity,order Repeated 7th
11.Tactile(Closest word)-SENSE OF TOUCH, tangible,material,physical,solid. Repeated 7th
12.Acrimonious (Synonym) : acerbic, harsh, scathing
13.Diaphanous : light, delicate, fine, See through(synonyms)

NMAT REPEATED QUESTIONS

14.Iconoclast (Closest word)a person who attacks or criticizes cherished beliefs or


institutions. Rebellious Repeated Nov 7
15.Umbrage (Synonym) : Offence ; annoyance
16. Chasten: Pure repeated 7th,8th,9th,9th(4:30)12 th
18. Parsimonious - synonyms:mean, miserly, niggardly, close-fisted, penny-pinching,
cheese-paring, ungenerous, penurious, illiberal, frugal (Penurious (Ant: Lavish) came on
11th 2 PM)
19. Bohemian- A descriptive term for a stereotypical way of life for artists and intellectuals.
According to the stereotype, bohemians live in material poverty because they prefer their
art or their learning to lesser goods; they are also unconventional in habits and dress, and
sometimes in morals. Repeat on 9th an on 11th (2 PM) repeated on 14th afternoon. Ans
(antonym): Conservative
small-town, inward-looking, limited, restricted, localist, conservative.
20. Aberration or Aberrant- Or Abhorrent (syn) : anomaly, deviation, dive Repeated 11th
Nov(4:30)
23.Venturesome-daring,bold,willing to take risks
24. Senile- (of a person) having or showing the weaknesses or diseases of old age, especially
a loss of mental faculties.
25 SOUSE - Synonyms alcoholic,alcoholic, alkie (or alky) [slang], boozehound, boozer,
dipsomaniac, drinker, drunkard, inebriate, repeated on 8th, 9th(4:30) 12th-11.30(synonym).
Options were different from here
Antonym - teetotaller, non-drinker
26 Mendacity : Syn : Dishonest [(Came on 11th 2 PM) Repeated from window 1], Repeated
11 Nov (4:30)
27 Obfuscate: Difficult to understand, confusing [Repeated from earlier windows, came on
11th 2 PM]
28 pusillanimous 12th

NMAT REPEATED QUESTIONS

Antonym - brave, courageous, daring, dauntless, doughty, fearless, gallant, greathearted,


gutsy, hardy, heroic (also heroical), intrepid, lionhearted, stalwart, stout, stouthearted,
valiant, valorous

NMAT REPEATED QUESTIONS

NMAT Window 4
Words :1. Insipid

tasteless, weak

2. Puerile

childish

3. Capricious

sudden change of mood or expression

4. Vindicate

justify, clear someone of blame

5. Bright

light, shine (antonym-dreary/dull)

7. Vacuous
intelligence; mindless

having or showing a lack of thought or

8. Capricious

unpredictable

9. Execrate

to dislike,criticise /declare evil, abhor

12. Exacerbate

to make worse

13. Occidental

relating to the countries of west

15. Malinger

pretend to be ill so that work is avoided

16. Guile

craft, cunning, slyness

17. Salacious

lustful

21. Bliss-

happiness

22. Sartorial

style of dressing/relating to tailoring

25. Accomplice
subordinate

person helps another in a crime or wrongdoing as a

22. Grandiloquent
23. Appease

extravagant, pompous
pacify or placate(someone) by acceding to

their demands
26 Abetter
27. Gargantuan

Accomplice
massive, huge

NMAT REPEATED QUESTIONS

28. Cantankerous

bad-tempered, argumentative

29. obloquy

strong public condemnation

30. Surreptitious

kept secret

34. Vilify

spread negativity

35. Onerous

burdensome, heavy10.

10. Laconic (antonym)

verbose, loquacious, talkative

11. Bright (antonym)

dreary (dull)

31. Malevolent(antonym) -

benevolent

32. Denial (antonym)

acceptance l

33. Fallacious (antonym) -

true, correct

34. Adjudicate -

resolve,judge, settle, arbitrate

35. Cicatrice -

Cicatrix, scar of healed wound

36. Inimical-

pernicious, destructive; hostile, deleterious

37. Repudiate ( antonym)

acceptance

Focus: Verbal Ability Questions


ANALOGIES
1. Tranquilizer: Sedation :: ice:cooling
2. Determination:Vacillation:: ( They are Antonyms)
3. Evanescence: Ephemeral (They are synonyms; Meaning - lasting for a very short time.)
4. Intractable: Manage (Antonyms)
5. Colloquy: Soliloquy::Monologue:Dialogue
6. Sycophant :adulate::Skeptic:Query

NMAT REPEATED QUESTIONS

NMAT Window 3
ANALOGY
1.Pedagogy: children
2.Claustrophobia: fear of confined space 12 th ans is acrophobia height
3. Thermometer : temperature I guess answer is calorimeter: heat
4. Steel:durable Ans:numbers:countable.
5. Villanelle:Poetry::?? Asked to pick out similar relationship from options.. mosaic:art
correct answer repeated on 8th mosaic is ans 12 th 14th too
RCS

NMAT REPEATED QUESTIONS

NMAT Window 4
Q30) If CONCENTRATE is written as ANLBCMRQYSC then what is ELIGIBLE written as
A) CKGFGAJD
(Malevolent, Fallacious, Cicatrice, Inimical )

NMAT REPEATED QUESTIONS

NMAT Window 2
1. Grammar was very easy don't remember the whole sentence
Except one machine in the factory..whats the error here!? Solution: Except For
2. He used to walk 10kms everyday despite of being 70 yrs old. Solution: Remove of
What is the error in question-2 used? Despite of is wrong. It should be just despite.
It should be despite being 70years old
3.

We can see the river the runs through the village..

What is the error in question-3 the ? That/which runs not the


METAMORPHOSIS by FRANZ KAFKA
It had an introduction to the 'metamorphosis' by the author, then how it is complex,
profound and realistic as much as it is unrealistic. And then the problems which the author
faced and how his family changed, then all these themes were reflected in the story with
the protagonist (the bug) , how loss of control affected the main character as well as the
author, and how alienation and isolation affected him, plus he had tuberculosis which
required isolation, main themes qas however the changes in the family under pressure and
unfortunate circumstances.
Why do we need to understand the authors mind (it was one of the last statements in the
first paragraph)
1. To know the history
2. To get the deeper understanding the story(ans)
3. To understand the parallel development in story and reality
Main theme of the Metamorphosis
Options:
Alienation
Isolation
Family(ans)
Change
Loss of control
What best describes metamorphosis
Options had Some random combinations of simple, complex, profound,ambiguous,
Kafkaesque, hidden
Title
1. An overview of the themes of metamorphosis (Ans)

NMAT REPEATED QUESTIONS

2. A bugs life

Odyssey and conflict (Came on 28th also)


nPlease try to search for keywords from RC whatever you remember, you might find the
source of RC on google. If you get something related, please post the link here.
2nd Nov RC on Odysseus life- 1. What is the tone of author?
A. Neutral B. Critical C. Argumentative D. Judgemental E. Donot rem(ANSWER)
2. What is the central theme?
Ans-Life is both search and realization
3. Why was Ramayana topic discussed? (answer)

2Nov.- 1) Cloud seeding technology. pretty easy one. (repeated on 4th Nov)
2) on lyrical ballads by Samuel Coleridge and william wordsworth (repeated on 4th
Nov)
FIB question (Nov 4)
(Had 4 blanks) it was on some corporate communication
1.
2.
3. Contemporary and formal language
4. Convivial cheers
Rc on non stop invention of trains by china
---> ANALOGIES
Earth: sun ::
Proton: neutron
Electron: nucleus
/Ans: electron: nucleus repeated 27,28thoct repeated 2nd Nov ,3rd nov,4th Nov
Fortification:attack ::
Ans: inoculate: disease - - > repeated 27th---->> Repeated on 28th

engross: forget
ans- absorb:distract
Repeated today 27 oct
REPEATED 28TH OCT, 2nd nov
Anthology:poem :: (2nd Nov)
Aesthetic : Creative

NMAT REPEATED QUESTIONS

Appraisal:praise was the option but not the answer! What is answer then ?????
27 Oct-insipid:interest -antonyms
Repeated 2nd Nov, don't remember the answer
ODD ONE OUT:
A. Orthodontist
B. Orthopaedist
C. Mycologist
D. Podiatrist
Answer C. 27th October
Mycologist
REPEATED 28TH OCT , Repeated 29th. Repeated on 2nd Nov, but the question was of some
other form.
Noun:a mycologist who specializes in the study of fungi.
1]ORTHOPEADIST-BONES 2]NEUROLOGIST-NEURONS
3]CHIROPODIST- FOOT
ANS-2
Yes as rest 3 are doctors in different fields and study parts of body.
Repeated today
Q. How many alphabets in REFERENCE have the same number of letters between them, as
in the eng dictionary?
Ans.Final Ans is 3.
3- EF FE EC (note: Zero alphabets in between is also considered as a valid condition)).Guys
what is the correct answer for this question ??? If it is 3, is it correct to check from the
reverse side too, like FE and EC?? Yes. Thank You!
Ans is 3 only. (Repeated on 4th Nov)
Q. Similar questions for MITSUBISHI (Nov 4)

NMAT REPEATED QUESTIONS

Quant Window 1 NMAT


Q1. Find remainder- (50^51)/53
Ans - 35
50^52 mod 53 = 1
50 50^51 mod 53 = 1 = 53k+1
-3 50^51 mod 53 = 54+++
50^51 mod 53 = 54/-3= -18 = 35
If it was 50^52/53, then the
answer is 1 (Fermats theorem)
Ans is 35 only!
(For 35)
Use the remainder theorem...you can google it for the procedure.
50^52 mod 53 is 1
So, write as 50^51*50 mod 53 = 1
1 can be written as 54
And 50 as -3
So, you will get -18 hu
50*(50)^51/53
The product of 50 n 50^51 shud come in the form of 53k+1
Since 50/53 is 50
So 50* something which should give 53k+1
50*35 = 1750 which is 53*33+1 = 1749+
Refer - https://www.quora.com/What-is-the-remainder-when-50-51-is-divided-by-53
Q 2 ) A person bought 864 articles and sold 800 of them for the price he paid for 864
articles. He sold the remaining articles at same price per article as the other 800. The
percentage gain on entire transaction is ?
Answer - 8%
Let cost of 1 article is 1 rupee. Therefore cost of 864 is Rs. 864.
Now, S.P of 800 is 864.
Cost of remaining = 64*(864/800) = 69.12
SO Profit = 69.12/864 = 8%
Ans - 8%
(64/800 * 100 = 8%)

NMAT REPEATED QUESTIONS

Let CP of 1 item be Rs1


Then SP of 800 articles=CP of 864 articles so SP of 800 articles= Rs 864
SP of 1 article is Rs. 1.08
Left 64 articles are also sold for same price
So total 864 articles were sold for 864*Rs1.08= 933.12
Profit= 933.12-864/864*100=8%
Let CP of 1 item be Rs1
Then SP of 800 articles=CP of 864 articles so SP of 800 articles= Rs 864
SP of 1 article is Rs. 1.08
Since gain % is asked, hence it can simply written as (1.08-1)/1*100 = 8%
Let the CP be x and SP be y
From the question: 864x = 800y => y = 1.08x
% profit = (y-x)*100/x
= (.08)*100 = 8%
Q 3) (101^4-1)....find the largest factor which is not the no itself.?
Answer- 10202 (I think this is the right answer)
k=(101^4-1 )/2=(104060401-1)/2
=52030200
Q 4) A shopkeeper marks his good at 60% above the cost price. He allows a discount of
12.5%. Also he cheats 14.28% while purchasing and 20%i selling and gives one article free on
purchase of 15 articles.
Find the profit/loss in whole transaction.
Ans :80% profit.
Cp=16*100*100/114.28
Sp=1600*()*1.2*15
Please give the correct approach for this ques ??
Take cp to be rs 100. Take into account all the gains and discount shopkeeper gives. Then
multiply cp with 16 (1 article free) and s
p with 15 to get profit %.
Better approach please?
Let CP = 100/article
so, MP = 160/article
After discount on MP
SP = 140/article

NMAT REPEATED QUESTIONS

Profits:
114.28% = 8/7
80 =
120% = 6/5
Final SP = 140 x(8/7)x(6/5) (it should be 6/5 in place of to get 192/article)
= 192/article
Since he gets money for only 15 b articles
Total SP = 15x192
But he pays for 16 articles while buying
Total CP = 16x100k
P = SP - CP
.
= 1280
P% = [1280/(1600)] x 100=
80%
Found the above solutions confusing, someone see if this is right,
CP per item - 100, MP - 160, SP 140.
Cost paid for 16 items = 16*100* = 1400
Money recovered for 15 items(last one free so zero) = 15*140*1.25 (20% cheating) = 2625
Profit = 1225 which comes to 87.5% of 1400.
Let CP of one article=x
CP of 15 articles=15x and plus one free =15x+x=16x
SP of one article= 1.6x*(7/8) *1.2*(8/7) =1.92x
SP of 15 articles= 1.92x* 15=28.8x
profit= 80%
Q 5) X and Y move from point A to point B. X starts moving 8 minutes after Y but reaches
point B at the same time as Y. If both of them start from opposite directions at the same
time, then they meet after 3 mins. In how much time can X cover the entire distance?
Case 1: When they are moving in the same direction.
D= Sx(t-8)=Sy(t)
So Sx=D/(t-8) and Sy=D/t
Case 2: When they are moving in opposite directions
Let them meet at a distance of m from A.
So m=3D/(t-8), and (D-m)=3(D/t)
Add both
D=3D/(t-8) + 3D/t
D will get cancelled.
Solve for t.

NMAT REPEATED QUESTIONS

You get t=12 and 2.


But you have to find t for X, which is t-8. Hence 2 isnt possible. So t-8 will be 4
Ans. 4 mins
Let total distance be D
Time of y=t time of x=t-8
Speed of y=D/t speed of x=D/t-8``````````````````````
Combined speed={D/(t-8)+D/t}````=> D(2t-8)/t(t-8)
From second equation we get (when approaching from opposite ends)
Total distance/combined speed of 2=3min
D/[D(2t-8)/t(t-8)]=3t(t-8)/2t-8=3
Solve for t
Answer is 4
t^2 8t = 6t 24 ni
t^2 14t + 24 = 0
t = [14+rt(196-96)]/2 and [14-rt(196-96)]/2
t= 2, 12
Time for X=t1-8minutes
12-8=4
Q 6) Mixture A of 738 litres has only chemical A and water in the ratio 7:2. Some amount of
water is added to mixture A to make the ratio of chemical A to water as 7:3. This amount of
extra water added is equal to the quantity of mixture B. Mixture B contains chemical B with
water at 10. How much water should be added to mixture B to increase the percentage of
water to 20%.
Ans - 10.25
Amount of water added in mixture A to make it 7:3=82 litres
10% water of 82=8.2 litres
Explanation:
10% of 82 is water, 90% is chemical B.
In the new mixture chemical B is same as old mixture. Thus 80% of new mixture is tu
chemical B ( 73.8 liters) thus 20% is 18.45 liters. Therefore,
18.45-8.2= 10.25 liters./
Or use the formula (WHAT IS C AND V)?

NMAT REPEATED QUESTIONS

C1v1= c2v2
.90*82=.80*v
V final = 92.25
Thus, water added 92.25-82=10.25
Q 7) A bee and an ant are present in one corner of a hollow cube. Both of them want to get
to the other extreme corner of the cube. The bee can fly it's way to the destination point
while the ant can only trace a path along the edges of the cube. What is the difference
between the distance covered by the ant and the bee if the side of the cube measured
10cm.
Ans 30-10rt3 (isnt this answer wrong??) {10(rt5 - rt3 )}
The question says that the ant moves through the edges of the cube. It doesn't say surface
of the cube. If the ant was moving through the surface the cube then the minimum distance
covered by the ant would've been 10rt5.
In this case, however, as the ant needs o travel through the edges only, it needs to travel
through 3 edges, traveling a distance of 30.
So the answer is correct 30-10rt3

Yes, body diag for cube is rt3* side. Therefore answer should be 30-10rt3
Correct me if i am wrong
Rt(10sq + 10sq) = 10rt2
Rt*(10rt2)sq+ 10sq)+=10rt3(whats the final ans? Please confirm.)
Root3=1.732
30-10root3
30-17.32
12.68
Don't be afraid if 30-10rt3 is not given in the option, they may have written it in more
simplified form
There is nothing mentioned about the shortest distance. It just says difference in the dist.
Correction- Distance for ant = root( (10+10)^2 + (10)^2) = root(500)= 10 root(5)....

NMAT REPEATED QUESTIONS

Q 8. A man receives Rs 600 as interest on principal when he deposited in a bank on simple


interest for 2 years. His friend told him that he would have received Rs650 for the same
principal if he had deposited it as compound interest for 2 years at the same rate of interest.
What is the rate of interest?
Answer = 16.67%
****Approach please: SI for first year = 300rs; CI for first year = 300rs; SI for second year =
300rs; CI for second year = 300+ interest on First year CI; So as per given 50rs is interest on
300rs and hence 50/300*100 is interest rate.
Solution: 600=p*r*2/100
pr=30000
50=30000*r/10000
r=16.6%( using the formulas)(for 2 years time period CI-SI= (p*r*r)/(100^2)
(SIMPLER APPROACH)
We know si remains same throughout
SI CI
X
X
X
.
---------------600 650
2X=600
X=300
SI
CI
300 300
300 350
-------------600 650
SO 50/300 * (100)
16.66%
Q 9)A man bought one motorcycle & gold (product) for 180000 & 160000, gold appreciate
@25% p.a & motorcycle depreciate @20%p.a. After three years he sold it and deposited the
profit he earned in a bank with 12% simple interest. How many years will it take to change
this amount to original amount(This kind of question was there)
Ans? According to incomplete Q/s 43yrs. My ans is coming as 83. Please check someone. Is
the right answer
(Data seems to be inconsistent. Anyone who can correct the question.)
I am getting 35 years.

NMAT REPEATED QUESTIONS

We can find what amount he ll have after 3 years, but without knowing profit % or SP , it is
not possible to solve it further
Q10- A tap can fill a tank in 6 Hrs and another one can fill in 12 hours. There is a leak at
distance from the bottom of the tank and the leak takes 24 hours to empty of the tank.
Then if all the taps and leak is open then how much time will it take to fill an empty the
tank.
Ans- 4.42 or 4 hrs and 25 min.
Exact ans is 4 hrs 25 min. and 42 seconds.
Ans:_ 4 hours n 36 minutes This should be correct answer
I think solution below is perfect.. 4.42 is the answer.. 4 hours 25 min 42 sec.
Correct 4.57 hr = 4hr 34min

**approach please**?? My answer is 4.6


I am getting 4.4 too
Getting 4.55
Ans : 4.42
Ans- 4.6 Correct answer {4 hours 36 minutes}
Check 3 solutions below and check which seems right
Answer: As 1 hr work: 16.66%
Bs 1 hr work: 8.33
Cs negative work: 3.11
Suppose capacity = 100 , so hole is at 25 from bottom.
Now to fill till 25lit no hole is present only A and B work so in 1 hr 25% of work is done
(16.66 + 8.33).
Now from this pt onwards A,B and C will work so in 3 more hours 65.64% more work is
done.
In all in 4hrs 90.64% is done. Now more 9.36% of work is done in 0.42 hours
So total time is 4.42 hours.
Let us take 48...efficiency
of tap 1 =(48/6)= 8
efficiency of tap 2 =(48/12) = 4
efficiency of leak =(36/24) = 1.5--{ of 48}
Now when it is empty for the 1st hour both tap will fill 8+ 4 =12 units(as the leak is at 12 litre
mark--( ))
For the second hour onwards it will be 8+4-1.5= 10.5 units=>12+10.5=22.5
Third hour=22.5+10.5=33

NMAT REPEATED QUESTIONS

Fourth hour = 33+10.5=43.5


Now if 10.5 units are filled in 60 mins then 4.5 (48-43.5) units will be filled in 60/10.5*4.5 =
25 minutes65
Therefore 4 hours 25 minutes or 4.42 hours
Or
Let tank be of 96 litres
Tap a fills 16 in 1 hr
Tap b fills 8 in 1 hr
Tap c empty 3 in 1 hour
Sab ko chalao - net 21 litre in 1 hour--{ab nhi chalenge ek saath->For the first hour c wont
work as the leak is at distance from bottom. 1st hour me 24 subsequent 21}
Time - {--96-24/21=>3.42+1 hour=4.42}
----------------------------------------------------------------------Let the capacity of tank be 24(LiCM 0f 6,12,24)
A does work of 4 units per hour
B- 2 units/hr
C-1 unit/hour
For of the tank ==>24/4=6
Work is done by A and B
4+2=6
Time required =1 hour
From now .. C comes in picture
Hence net work will be A+B-C=4+2-1=5
Time required = Work left/ new wok= 18/5=3.6
Total time = 1+3.6 =4.6== 4 hour 36 mins
I think last approach is correct.
I think 4 hours 36 mins is wrong. Because in your approach As and Bs efficiencies are
considered with respect to the entire volume of the tank while Cs is only being considered
with respect to of the volume which is wrong.If A and B takes 6 hours and 12 hours to fill
then C takes 32 hours to empty the tank. Therefore we need to find the LCM of(6,12,32)=96
and solve.Ans- 4hrs 25 mins.
> Suppose at start the tank is empty, Pipe A & B will operate alone upto of tank and after
that A , B & C will operate for th of the tank.
Pipe A fills whole tank in 6 Hrs , so 1/4th of the tank will be filled in 3/2 hours
Pipe B fills whole tank in 12 hrs , so 1/4th of the tank will be filled in 3 hrs
So 1/4th of the tank will be filled by both the pipes in ( + = 1 Hr)

NMAT REPEATED QUESTIONS

Now th of the tank is full and now the leak C will come into action
Pipe A fills whole tank in 6 Hrs , so 3/4th of the tank will be filled in 9/2 hours
Pipe B fills whole tank in 12 hrs , so 3/4th of the tank will be filled in 9 hrs
Pipe C empties 3/4th of the tank in 24 hrs
Collectively they will fill the tank in ( 2/9 + 1/9 - 1/24) = 7/24 , reciprocal =24/7 =3.42
Hence total time taken is 1+ 3.42= 4.42. Hope this explanation clears

Q 11). If X and Y is in the ratio of 5:3 then which of the following cannot be the sum of X and
Y. (Options were 96, 112, 128, 100)
answer - 100
96 = 5*12 + 3*12
112= 5*14 + 3*14
128= 5*16+ 3*16
Or simply the numbers should be divisible by 8 i.e. (3+5)
Q 12. The difference between 25 % and 40% of a number is 135. Then what will be 80% of
the number?
(Answer: 720)
(40-25) = 15% = 135 . Hence 100% = 900 . Hence 80% = 720 .
Q 13. A dealer makes a profit of 25% on an item. If the selling price increases by 15% and
the buying price decreases by 15% then what will be the difference of the profit percentage
as compared to before?
(Answer : 44.11 )
Earlier :
= 100, SP = 125 (profit 25%)
Later : CP=85, SP=143.75.(So, Profit would be 69.11%)
69.11 - 25 = 44.11
Q 14. An organisation has 45% male employees of whom 60% are officers. If there are a
total of 60% officers in the organisation then how many female non officers are there in the
organisation?
.(Answer -- 40% )

NMAT REPEATED QUESTIONS

(2/5 as per options in the paper today) How 40%? (I think it is for total non-officers)
Assume 100 people.
Male (27 officers + 18 non-officers) , Female (33 officers + 22 non officers) - Answer should
be 22 % ??
If it is asked % of female employees who are non officers then ans is 22/55 * 100 = 40%
If it is asked % of female non officers then it will be 22/100 * 100 = 22%
.Q 15. A regular hexagon has sides of 20 m. On each of the corners of the hexagon a horse is
tied with a rope of 7m. Then find the area in the hexagon not grazed by the horses.
(Answer : 600rt3-308 )
Area of hexagon -6(area of sector)z
Area of hexagon =[3(rt3)*(s^2)]/2=600rt3
Area of 6 sectors - (22/7)*7*7*()*6=308
Therefore, 600rt3-308=731.23
Q 16. A coordinate geometry question in which the two coordinates of a vertex of a triangle
were given and the third needs to be found. It is also given that the triangle is equilateral. (Think of a GRAPH to visualize.)
(y2-y1)^2+(x^2-x^1))^ =other 2 sides
Find the length of sides using the two vertices that are given and then check from the
options to get the other two equal sides. All sides equal since the triangle is equilateral.
d(P, Q) = (x2 x1)2 + (y2 y1)2
Q 17. A factory has 16 machines which can make 5000 sheets in 10 hours. If these 5000
sheets are to be processed in 40 hours then what would be the number of machines
required if the efficiency of the machines remain the same.
ANSWER IS 4.
USE THE FORMULA MD/W1= MD/W2
we get 16*10/5000 = x*40/5000
x=4 CONFIRMED

NMAT REPEATED QUESTIONS

Q 18. Ramesh and Yogesh invest in a partnership in the ratio of 4:5. After 6 months Jayant
invests in the partnership with the same amount as Yogesh. After a year the firm makes 20%
profit which is 92,000/-. Find the amount invested by Yogesh.
ANS. 2,00,000.
Total Investment = 460000 (R:Y:J= 4*12:5*12:5*6) ~ (R:Y:J=8:10:5) (explanation for this
step?)[HAVE EXPLAINED.SCROLL DOWN A BIT]
so, (460000/23)*10 = Yogeshs Investment.
How did you get 4,60,000?
well because let the his initial investment be x,the new gain is of 20% or 1.2x,but it is said in
the question that the profit is 92,000 i.e 1.2x-x=92,000 or 0.2x=92,000.Hence,we get
x=46,000 .Hope it helps. Thanks :)
Above ans is wrong
It dont ask for profit ..it ask for investment so ratio is 4:5:5
Total Investment = 460000 R: 131428.57, Y: 164285.7, J: 164285.7
No,investment period too needs to be taken into consideration.
Therefore,as 6 month calculation,
RAJESH:YOGESH:JAYESH = (2*4):(2*5):5 =8:10:5 [1st two are multiplied with 2 because 1
year= 2* 6 month
periods.]
Total investment made in the year = Rs 4,60,000. (as 20% =92000).
So Yogeshs investment is = 10*(460000/23)=2,00,000 [8+10+5=23]
Q 19. There are 10 Hollywood Movie Disks and 25 Bollywood movie disks on a table. Then
what is the probability of picking up a Hollywood movie disk?
Ans. 10/35= 2/7
Q 20. Mahesh has to type 10 pages, he can type 25 lines in X mins and each page has 100
lines. Then in how much time does he type 100 lines if he takes a break of 10 mins in
between?
(Doesnt the question mean that for typing every 100 pages,Mahesh takes a break of 10
min.So what is the total time taken Nby him to type 10 pages of 100 lines each?)
Yes, I think it should be 20(x+5) (Please confirm).
Answer: 2(2x + 5)

NMAT REPEATED QUESTIONS

25 lines X minutes
100 lines how many minutes?
100*X/25 = 4X plus 10 extra minutes 2(2X+5)
Q 22. A question on true and false where there were 4 statements on properties of
quadrilaterals, Rhombus, Trapezium, Parallelogram were given and one has to say whether
the statements are true or false.
Q 23. If product of two integers P & Q is 42 then what is the value of P?
a. P is an odd number and larger than Q
b. P is in the form of 3K
Correct : Neither of the above statement / yes both is true only for positive integers.
(i think using both the above statements should be the answer as p can be = 21 i.e 3(7) and
q=2)
Neither statement is sufficient as even after combining, we have two possible solutions
P = 21, Q = 2
P = -3, Q = -14
Q 24. What is the perimeter of triangle a,b,c?
a. Abc is a right angled triangle
b. The sides containing the right angle is of 8cm and 6cm.
Ans.ONLY B is sufficient as the word right angle is mentioned in statement B also.
Q 25) A&B start from P&Q respectively and meet after some time. And complete their
remaining distance in 2hr n 3hrs respectively.find ratio of their speeds.
Answer: root 3 : root 2
Please explain above??/ how root 3 : root 2?? (you can remember it directly . It's always the
root of reciprocal of time ratio)
Let the guys meet at point K.
Time to reach a point K, t = PQ\(a+b) where a and b are speeds of A and B respectively.
Now A has to travel remaining KQ while B has to travel KP.
KP = Distance travelled by A in time t calculated above and similarly for B.
KP = a*t = a*PQ/(a+b), KQ = b*t =b*PQ/(a+b)
As given in question, nd a*t/b = 3b*t/a = 2 a
So eliminating t => a:b = sqrt(3/2)

NMAT REPEATED QUESTIONS

Q 26) x, y ,z are in GP. Is the common ratio negative?


Ans. statement one can answer and hence is sufficient but statement 2 isnt.
Statement I : x>y>z
Statement II : x and z are negative
Statement 1 is sufficient. Take cases. 4,2,1. -1,-2,-4. 2,-1,0,5(does not satisfy condition)
hence Statement I is enough. Thus X>Y>Z is maintained only if it's a +ve ratio
With statement 2,two possibilities are there, 1.when y is positive and so,r is negative and 2.
When y is negative and so,r is positive..Sign of r varies.
Q27) Sigma(3+2^r)=?, when r=1,2,....,10
Ans = 2076 Confirmed
30 + 2^11 -2 = 2076
Q28) Pipe 1 can fill a tank in 16 hours. Pipe 2 can fill it in 8 hours. They
are
kept
on
on
and
l off alternatively for 1 hour each. When will the tank get filled?
Ans.11 Hours (confirmed)
(I think 11 hours) If we assume total tank capacity to b 32litres then the effic
iency of first pipe will be 2 and 2nd pipe will be 4. Now if we calculate den
in 10 hrs alternatively if both pipes work then it will be 30 liters and den l
ast 2 liter will be filled by first pipe. So in total 11 hours
It is 11 hours. Confirmed.
How. It's not given which one was started first ? - exactly. Without knowing who starts first
we cant say!!
But the question starts with Pipe 1 and Pipe 2 being used alternately. So we can assume it is
in
that order only otherwise they would have specified otherwise.
It is 11 hours if pipe 1 is started first and 10.5 hours if pipe 2 is started.
Correct Soln- For this Question, the answer is 11 hrs.
With LCM method,
Pipe A -----16 hrs
Pipe B-------8 hrs
Total units 16 units . (LCM of 16 and 8)
Hence Pipe A fills 1 unit in 1 Hour while Pipe B fills 2 units per hour.
Since the pipes are opened and closed alternately,
In First hour 1 unit is filled by Pipe A ,
In Second hour, 2 unit is filled by pipe B,
So in 2 hours , 3 units is Filled.
We have to reach close to 16 units..so lets multiply both sides by 5 ..
So in 10 hours 15 Units is filled ..now in Next hour 1 unit is filled by Pipe A ..so total units
16 units filled nad total time ..10 hours + 1 hour = 11 Hours.
(Note- It looks lengthy coz I have tried to explain, method is very simple. )

NMAT REPEATED QUESTIONS

Q 29) Logbase x(logbase y(log base 2 z) =0 for what value will give z
Answer=2^y correct
Remove the first log by taking x to the other side. x^0=1.
Then remove y in the same way. y^1=y
Remove 2 in the same way, 2^y=z
Q 30) If x is highest power of 30 in 50! How many factors does (x+3)! Have.
Ans: 4032
calculate the highest power of 30 in 50! Which is the highest power of 5 in 50!
Therefore, x=12
X+3=15
Now find the prime numbers less than 15, ie, 2,3,5,7,11,13
Now find highest power of each of the prime factors in 15!
highest power of 2 in 15! = 11
highest power of 3 in 15! = 6
highest power of 5 in 15! = 3
highest power of 7 in 15! = 2
highest power of 11 in 15! = 1
highest power of 13 in 15! = 1
now apply the formula for finding number of factors:
(11+1)(6+1)(3+1)(2+1)(1+1)(1+1)=12*7*4*3*2*2=4032
Guys is 4032 the confirmed answer? YES
Q 31) A train is moving from B to Z with 50 km/hr and then back with speed 60m/s. What is
the avg speed?
Ans.22.55 m/s or 81.20 Km/hr
Approach - Convert 60m/s into km/h. Its 216km/hr.
Let B to Z distance be x. Then Z to B is also x. Total 2x. Divide 216(60+216) by 2x! We just
need the average speed!---> Please google it.
When the distance is same the average speed is the harmonic mean of both the speeds, if in
m/s then the ans would be 22.55 m/s or else in km/hr it is 81.20.---exactly!
2XY/(X+Y) will be the ans.
It should be 81.20km
hr 2xy/x+y is average speed for same distance ..Just apply the formula . 2xy/(x+y)(Harmonic
Mean)
( there is no option in 90s, so guys who got the ans in that you are doing something wrong)
Here option that is ans is 81km/hr. This was the closet option rest were like 216, 100
km/hr.

NMAT REPEATED QUESTIONS

Q 32) There are total 1000 people 790-scooter, 195-cycle and 60 none. Both?
It should be 45, as the number of people who liked at least one is equal to 1000-60 = 940
Therefore (A U B) = 940 = 790+195-x
Ans 45
Q 33)There are 100 balls, 60 yellow and rest green. Half are glass and others plastic. There
are 25
yellow glass balls. A ball is picked and it is green, what is the probability that it is plastic?
ll
Conditional probability has to be applied here. (15/100)/(40/100) = .
Yellow.
Green
60.
40
Glass Plastic. Glass Plastic
25.
35 25. 15
Prob = 15/40 = 3/8
Answer -
It should be 3/20(correct), as the total green balls which are plastic is 15 and the total no of
balls is 100, Probability = 15/100 which is 3/20.
Can someone please explain this question properly?
Total : 100
Yellow: 60
Green: 40
Half are glass and others plastic. There are 25 yellow glass balls??
ANS: 100 balls,
60 yellow- Glass 25, plastic 25
40 green - glass 25, plastic 15
Its given that ball picked is green. Hence its conditional probability. So, total sample spaces
is 40 and not 100. Plastic green balls are 15. Hence 15/40 or .
34)XYZW is a quadrilateral circumscribed by a circle. Angle YXZ=70, Angle YWX=40, (I think)
WX
was given as 4 and YZ=7. Measurement of one the diagonal was asked?
Irrelevant //whats with this question ????WE can calculate it by cosine formula this sum .
No need of any cosine formula. If the diagonal YW was asked for, it'd be 7. As YWZ is an
isosceles triangle. If the diagonal XZ is required, it can't be determined with the set of
informations

NMAT REPEATED QUESTIONS

Both the diagonals are equal in length as can be found out after getting all angles. So, both
diagonals measure 7 (as explained above).
Q 35) There are 100 balls out of which 60 are green and rest are black. Out of all balls, half
and plastic and half are glass. Out of the green balls, 1/5 of them are plastic. If a ball is
picked randomly, what's the probability that is is a plastic ball given that it is green in
colour?
ans-
Q 36) X^0.5* x^0.25*... Till infinity
Answer : x -- how??
The power of x is in gp so r is 0.5.Now use the formula for infinite progression
= X ^(0.5+0.25+...) = X^1 = X
Using g.p. Infinite sum formula, shouldnt the ans be 2x??
Method: a/(1-r) = x/(1-0.5) = x/0.5 = 2x
No. here a is 0.5 . So it would be 0.5/(1-0.5)=1
Q 38) A ques was number divisible by 153 of the following options .
Ans was 92565
Q 39) c - SOLUTION
Q37) A and B are two candidates in elections , A got x% votes and B got (x+20)% votes . 20%
people did not vote . Find x
Answer: 30
100-20 = 80%
2x+20 = 80
X=30
(is repetition allowed or not? Out of 4 vowels 2 can be chosen in 4c2/2!*2 Ways. if
repetition is not allowed then (4c2/2!*2*6*5*4 )*4 will be the hi answer else
(4c2/2!*2*8*7*6) *4 will be the answer)
2880?
Answer: 1152
Am getting 1776 PLEASE CONFIRM????? AS PER SOURCE THERE WERE NO OPTIONS MORE
THAN 1000.HOPE THAT HELPS
OU_ _ _ =4!*6c3*2!
IO_ _ _=4!*6c3*2
OO_ _ _=4!*6c3
UI_ _ _ = (If OO both are taken 4*24 = 96

NMAT REPEATED QUESTIONS

(If O is used = 4!*4c2*2! (If no O is taken = 4!*4c3*2!)


Form words where no vowels are together. Vowels can be arranged in 4!/2 ways. OUIO
SLTN
Fill gaps in 4P3 ways last consonant can have 4 locations
So 12* 24 * 4 = 1152
Total ways = 8!/2
So answer is 8!/2 - 1152 = 19008
Correct answer-1152/2880
What is the correct answer for this???
Q 40 .. FDB Builders was awarded the contract to construct a bridge. The company
employed 100 workers to finish the work in 120 days. When four-fifths of the work was
completed in 80 days, the company wanted to reduce the number of workers. How many
workers can be let go without affecting the completion schedule of the construction of the
bridge? (not a nmat question its official mock ques)
Answer. 50
Total Workers =100, Total days 120
Work done in 80 days = 100x80 =8000 man-days
This is of total work
So, Total work = 5/4 x 8000 = 10000 man-days
Work left=2000 man-days, No. of days left =120-80=40
Hence no. of workers required to complete that work= 2000/40 = 50
So 50 workers out of 100 are required to complete the remaining work in 40 days
Therefore, 50 workers can be let go.
(W1/W2)=(M1D1/M2D2) Here W1= , W2= , M1=100 D1=80 M2=? D2=120-80=40
You will get M2=50 , so 100-50=50 workers can be let go
Q 41 A pump can be used for filling as well as emptying a tank. The capacity of the tank is
1,500 m 3. The emptying capacity of the tank is 10 m 3 per minute higher than its filling
capacity and the pump needs 5 minutes less to empty the tank than it needs to fill it. What
is the emptying capacity of the tank?
Answer: 60 confirmed
Emptying capacity = x
Filling capacity = X-10
(1500/x-10) - (1500/X) = 5
Solve. U'll get 60
Am getting quadratic equation x^2 + 10x - 3000 = 0 (the quadratic would be x^2 - 10x - 3000
= 0 solve it you get x=60 and x= -50
Solution please??
Ans.

NMAT REPEATED QUESTIONS

Let the filling capacity of the tank be x


emptying capacity x+10
Acc to statement
1500/x - 1500/x+10 = 5
Calculate x, its 60. (ANSWER)
The above quadratic equation gives x= -60 and 50. So the answer according to your equation
should be 50. Please check.
Yes but from this quadratic equation you get that x (filling capacity) is 50. So, the emptying
cap is 50+10= 60
CONFIRMED.

Q 42.Cost of an original diamond was Rs 96,000 . Ratio after the diamond being broke was
2:3:5. The cost is directly proportional to the square of its weight, Find the loss?
Approach : 96,000= 10^2*K (K=960)
2^2*K= 4*960=3840
3^2*K=8640
5^2*K=24000
Total Value of- the diamond that broke = 3840+8640+24000 =36480
Loss= 96,000-36480=Rs 59,520
IT SHOULD BE K^2 IN STEAD OF 2*K.i
Also,this shouldnt be in the power of 10.Isnt it? Ya right but if ratio of
weight is given
then it is right
An easier solution :
Current Valn is (2^2 + 3^2 + 5^2 )% = 38%
Therefore loss = 100-38 = 62%

NMAT REPEATED QUESTIONS

0.62 * 96,000 = 59,520


Q 43. If A is born on tuesday in 1996. What will be his age when his birthday will fall again
on tuesday?
Ans : CRUX: 6 or 7 depending on whether he is born before feb or after feb ; CONFIRMED
Today again this question came, september.
Is the answer 28 years or (6 or 7 depending before or after feb)
Q 44. What is the minimum no of terms required for a total between 1200-1400? PS: can
someone explain this question?
Minimum number of r required to form any number between 1200-1400
1,2,4,8,16...1024 = 11 terms ----how did you conclude that terms means GP terms with
r=2 ?
Any better approach ??
The series was given 3*9 + 6*14^2 + 9*16^2
Mostly the answer is 7.
Q 45.Diagonals of a rhombus are 24 cm and 10cm. Find its perimeter?
Answer : 52
Area = *product of diagonals
=*24*10 =120
now using pythagoras theorem find the sides.
Side=sq rt of (25+144)
Side = 13.
Perimeter = 4a i.e 4*13 = 52.
FORMULA FOR PERIMETER OF A RHOMBUS= 2* RT(D1^2 + D2^2)
Diagonals bisect at 90 degree so (12,5) which will give side as 13 directly no formula
required
Q 46). Sum of N natural number is 36/204 times sum of square of first N natural number ??
Ans is 8
(n(n+1))/2 = 36/204 * ( n(n+1)(2n+1)/6)
47. The ratio of children to adult going to zoo on saturday was 4:1. On sunday 20 more
people went to zoo, among which 2 were children. The ratio of children to adult going to
zoo on sunday was 2:1. Find the total number of people who went to zoo on saturday?
Answer: 85.
{(4x+2)/(x+18)}={2/1}

NMAT REPEATED QUESTIONS

Q 48. A is a working partner.B and C are investing partner. B and C are investing in ratio 2:3.
A leaves for 3 months. Total profit for the year is 10 lacs. B and C divide the profit as
per investment ratio and it is decided that A gets 50% of the profit. How much is the share
of B?
Options: 1) 2,00,000 2) 2,50,000 3) 3,00,000 4) 1,00,000 5) 1,50,000
Answer 2nd option= 2.5 Lacs.
> 10 lacs is profit for 12 months, so for 9 months profit will be 7.5 lacs. 50% of this is 3.75
lacs
Hence A will be paid 3.75 lacs out of 10 lacs. Remaining 6.25 lacs will be distributed among B
& C in ratio of their investments. This gives share of B= 2.5 lacs
50% profit of 7.5L (9months) ------How???? I am getting 1st option i.e. 2,00,000 <- me too
PLEASE CONFIRM?????
Even i got 2,00,000.(Solution given above)
Q 49. DS QUESTION
There are 5 balls of different color. They need to be arranged in a stack form. Determine the
sequence of the balls
A) green ball is on top. Blue ball is in between orange and yellow. White ball is in the last
position
B) the same data as A,only difference being that orange ball is one step ahead of the yellow
ball
B is sufficient. (G O B Y W)
AnsG
O/Y
B
Y/O
W
I think not sufficient.
I got this ques:
There are 5 balls of different color. They need to be arranged in a stack form. Determine the
sequence of the balls

NMAT REPEATED QUESTIONS

Between orange and yellow there is one red ball. Pink is on top and blue ball is in the last
position.
B) red is between orange and yellow. Above pink there is no ball and blue is at last and
orange ball is one above the yellow ball.
Ans is statement 2 is only.
Q 50. A shopkeeper earns a profit of 25% on an item. A boy purchased the item for 15%
more than CP and the shopkeeper paid 15% less than the cp while purchasing the item. Find
the difference in profit percentage?
Assume price of item =100
Boy purchased =115 =SP
Shopkeeper paid= 85=CP
profit= 30/85=35.2%~35%
Difference in profit in percentage = 35-25/25 = 40%=ans??
Shouldnt it just be 35.2-25=10.2%. Since they have asked different in Profit %, not %
difference in profit %. Please correct me if i am wrong?
Q 51. A shopkeeper marks the price of goods up by 60% and offers a discount of 40%.
Further he uses weights 1250 gms hi while purchasing goods and 800gms while selling
goods. Find his profit percent. ANS???? 50
I think =45??????
Assume = cp =100 so mrp=160 sp=96
For 1000 gm good price CP = 100 but he got 1250 gms=125 rs =25 rs profit
For 1000 gm good price SP=96 but he sold 800 gms = 76.8 rs =20 rs profit
Total profit = 25+20 =45
Profit percentage = 45*100/100= 45%?????[Depends on the interpretation of the
question,the wordings arent clear+
Someone please clarify this!?
Same ques but i got a discount of 20%. And i got profit % as 100% options were 50, 75,
100,150.
I think This approach is wrong
Answer should be 50%
Method:
Let assume 1 gram=1 Rs.
He buy 1250 gms for 1000 Rs.
So Price for 800 gms is 640 Rs.8,

NMAT REPEATED QUESTIONS

Bcoz he sell 800 gm instead of 1000 gms


Selling price is 1000+600(60% profit)=1600 - 640 (40% Discount) = 960 Rs.
So, CP=640 SP=960
50% Profit
By
Q 51. A sector of circle is folded to form a cone and inserted into a hollow cylinder of
volume(i don't remember the value) cubic meter. Height of cylinder is same as cone 14m.
Diameter of cone and cylinder is same. Find the radius of the sector?
e radius of cone.
Now in the cone slant height will be the radius of the sector however we fold it slant height
becomes the radius of sector.
So slant height=rt(h2+r2)
We can get the radius of sector this way
Please correct me if am wrong???
( IT WONT BE PI*r*r*h since it's Hollow cylinder. It'll be pi*(R^2 - r^2) *h
Someone please clarify this^?
Q 52 A person goes from one place to another at 50km/hr and further goes to another
point at 60m/s . Find avg speed
(What sort of a question is this? :/ )
Q 52.
Find angle in red.
I am not sure about the location of the given angles but these are the exact measurements.
If anyone remembers this question precisely, feel free to edit.----> The values given at those
positions are not possible. Both angles are subtended by the same chord at the
circumference. Hence, they should be equal.
If the chord is the diameter then it should be 90.
Q 56 X invested some T amount in 2 schemes at 5% CI. Scheme A for 3 yrs, Scheme B for 5
years. Investment in scheme A/B?
A/B = T * 5*f*f*f / T *5*f*f*f*f*f= 1/f^2 where f = (1+r/100)
So = 1/f^2 = 1/ 1.05^2=A/B = 400/441

NMAT REPEATED QUESTIONS

ANS = ratio of investment A/B = 400/441


Ratio of investment was not the question. The investment done by either A or B was asked=
aree amount should be given in que but somebody didnt mentioned so using the ratio of
a/b u can find investment done by either a or b=
For this que =iinvestment byA = 400 T/841
ISNT T THE TOTAL AMOUNT PUT INTERNET BOTH THE SCHEMES?
PLEASE CONFIRM?????
Q57 In a fair children to adult 4:1 on saturday. Next day 20 more people came of which 2
were children. Ratio became 2:1. How many people in fair on saturday?
Answer: 85
4x+2/5x+20=
12 x +6= 10x+40
x=17
Sat == 5x=85
-----------------------------------C/A= 4/1
Next day, C+ 2 / A + 18 = 2/1
To find: C + A = 5 A
On solving, A = 17
So, answer = 5 A = 85
Q 58 Length of platform A to B in ratio 4:3 ; length of train A 300m. Time taken by train A to
cross platform is to B in ratio 6:5 time taken by B is 30s. Find speed of the train in kmph ?
question seems little incomplete
Using the 2 ratios find length of platform B and time taken by train to cross platform A.
Using d/s=t find length of train from 2 eqn and subsequently find the speed. Ans was 54
kmph
My ans is coming as 90 kmph since length of platform B is coming out to be 450m , so speed
of train=(450+300/30)=25m/s = 90kmph (correct me if i am wrong!!!)0
30seconds is the time taken by train B to cross the platform not train A
Question not clear 30 s is for what?
What is the approach to this?????
Question not clear. Someone who remembers kindly edit the same
PLEASE CONFIRM?????
Guys these question was incomplete..the length of platform A was 300 nd B was was 225(as
i deduced) and time taken to cross A was 30 sec and so the speed comes out to be 10m/s
sec...which is 36kmph

NMAT REPEATED QUESTIONS

Q 59 Investment of A:B:C was mentioned in words so had to deduce. Total profit was given.
And then avg 5000 of B C D. Ratio A:D investment //can someone plz complete this question
??
Q 60. 1 dress stitching time by A is X hours and by B is Y hours , C takes as much time as
both A & B together take to make the dress. In what time 6 dresses will be made by B & C/
ANS. 62X+Y)(XY/
Assume for A x=10 hr and for B y=10 hr now for C = xy/x+y =5
b&c = 10*5/10+5 = 10/3 time taken by b&c for stiching 1 dress
For 6 dress = 6*10/3 =20 =interms of X & Y = 2X or 2Y
Wont it be 6(XY/2X+Y)??
Let LCM(A,B)= XY Units = Total work
Unit/hr A = Y, Unit/hr B = x . Unit/hr C=B+A = X+Y .. Unit/hr B+C = 2X+Y ...therefore days
by B+C = XY/(2X+Y) for 1
dress.for 6 = 6XY/(2X+Y) ??
Yes!
Q 61 Two pipes A and B are started at 9 am. A fills the tank in 24 mts and B takes 1.5 times
the time taken by A. When should pipe B be closed such that the tank is filled in 18 mins?
Ans 9.09 am ? HOW??
Ans. Let Tap Y be opened for M minutes and Tap X will be opened for 18 mins (9.18 -9.00=18
mins)
18/X + M/Y = 1 (WHOLE WORK DONE)
18Y + MX = XY
M= (XY - 18Y)/X
> {(1/24 + 1/36)*X} + {(1/24)*(18-X)}=1(Whole work done)
Q 62) Series 1 a=2 d=3 n=80
Series 2 a=3 d=2 n=100 .
No of common terms?
Can someone please explain this ?
Find last term of b-oth APs:
AP1= 2, 5, 8.239

NMAT REPEATED QUESTIONS

AP2=3, 5, 7.201
First common term = 5 (=a)
d= lcm(2,3)=6
Solve for n,
5+ (n-1) 6 <=201 (since 201<239, the terms wont exceed 201)
n-1<=32.something
Therefore, n=33
Q 63) X goes from A to B at 3 kmph and returns at 4 kmph. If he travels at 3.5 kmph both
sides he
completes the journey 5 minutes earlier. Find the distance travelled by X.
Ans. 14 km (Approach?)
One side distance is 7 km so should the distance travelled be 14km (to and fro) or 28km or
7km ?
14 was not an option. Distance from one side was asked. I was not sure of the question
earlier.
I am getting 7
D/3 + D/4= T--(1)
D/3.5 + D/3.5 = T-5/60--(2)
From 1 D=12T/7
2=> D [ 4/7] = T-5/60
12T4/7=T-5/60
T=49/12
D=12/749/12
D=7
Q 64) Geometry problem trapezium fig. Application of pythagoras twice . what was the
answer?
y was to be found out
25^2-7^2=24^2
So side was 24units
24^2+10^2= 676=26^2 ans 26
Q 65. A warehouse stores 345 ltrs of Chem A, 120ltrs of Chem B and 225 ltrs of Chem C.
Biggest size of the cask and in which it could be stored. (Values were different) /* Question
Picked up from Official Guide */
Approach- Find the HCF of the three numbers. That should be the answer. In this case, it
should be 15ltrs.

NMAT REPEATED QUESTIONS

Q 67. Highest power of 5 in 360!


> 88?????
Approach: 360/5 = 72, 360/25= 14, 360/125=2 (72+14+2=88)

Q 66) 40 people from the class cleared the maths test. 20 got negative marks. 10 did not opt
for maths and 15 did not write the paper. How many students were there in all? It was
somewhat like this. (Numbers were different)
Approach- Just add the numbers. 40+20+10+15= 85 in this case.
Q 69. LCM is 1534 of 4 numbers. Some 3 numbers were given and the 4th number had to be
found out. How many such values were possible? (Dont remember the values)
Answer: only 1 such value 1534=1*2*13*59 (if out of these 3 are given only one way can be
there for the fourth one)
Q71. A horse is tied with a rope of 7cms inside a rectangle of dimensions X*Ycms. Find the
ratio of area the horse could graze outside the rectangle to area that could be grazed inside.
(I think the answer to this question was 3:1)
Yes, its 3:1 CONFIRMED!
Q 72 The numbers 2010 are marked on a circle and numbers are circled starting from 1 after
every 7 numbers (1,8, 15 etc..). If this process is repeated, and stopped only when the mark
could be put on an already marked number, then how many numbers were
Because we can right 2010 as 7k+1. So every number after 7 number will marked then all
numbers are marked so
0 unmarked number.
If it ask for 2011 then answer should 1.
Q 73 Two bells ring at intervals of 25s and 15s and ring together at 3 PM. At what time did it
ring together before.
Approach?
Take LCM of 15 and 25= 75, therefore it happened 1 min 15 sec before i.e 2:58:45 pm.
Q 74. In how many ways can 8 students sit around two circular tables located in two
different rooms such that 4 students sit around each table?
8c4(Select any 4)*3! (arrange 4 on table 1)*3!(arrange remaining on table 2)=2520

NMAT REPEATED QUESTIONS

Q 75. The square of the sum of two non-negative odd integers X and Y is 16. What is Y?
Y could be 1 or 3. How??
When square of a number is 16,the number must be either 4 or -4.Since its giving that the
number is positive,thus sum of X and Y must be 4.Also,this 4 should be a sum of two odd
integers which in this case can only be 1 and 3.
Q.76 What is {Nr ! (Ms+1)!} /{(Nr -1)! (Ms )!}
Ans.Nr*(Ms+1)
Nr(Ms+1) : Let Nr=x and Ms =y,
x.(x-1)!*(y+1)*y!/ { (x-1)!y!} = x*(y+1)
Q 77.) 18, -12, 8 . is a GP find the term that is (- 256/243)
, 18 *(-), 18*(-)(-), 18(-)(-)(-)...............
18 ,-12, 8,- 16/3 ,32/9, -64/27 ,128/81 ,-256/243
> ar^(n-1)=(-256/243) , Here a=18 & r=(-), we get n-1=7 . Hence n=8
Q78) x is no. of trailing zeroes of 100! in base 7, x-10 is base of n! such that trailing zeroes
are 10...find least value of n
Somebody please share the concept. How to solve it
ans is 24
How to solve this??
Can someone please explain this
Q79) 2 teachers and 3 students should be arranged in 4 cars such that no student sits with a
teacher and the cars can be empty
Ans: 112?
Q 80) Mona and ajit and ten other students sit in a row. What is the probability that mona
and ajit has exactly 4 students between them
Ans: 10!*7*2/12! = 7/66
Q81) 3 guys m , r and b , can complete work in 20 30 40 days,If m and r work on first day
together and m and b on second day....

NMAT REPEATED QUESTIONS

They
And : 12 days
Q82) There are 7 ppl and 11 floors Each person gets on a different floor And only one
person per floor. What can be the different ways?
Is ans 1663200?
And: 11p7 =1995840
Q83) (500+17 )^xyz units digit,
I dont remember the exact values . However (517)^xyz ,it was 171
Check for the cyclicity of 7
Ans was 3 so 171=4k+3
Q84)On a circle 10 points p1 p2...p10 are marked Two lines intersect
And :126
Approach please?
Q85) Roots are reciprocal . Find relation p and q
Eqn px^2+ rx+4q=0
Answer?????? Is it p/q = 4?? Yes!
left un-marked at that instance.
ans : 0
I also got 0.
Q86) 63. other day two of machine A company has three machines A, B,C. On one day one
machine A, two machine B, 3 machine C worked for 96 hours. On an A, three of machine B
works for 80 hours. On another day two machine A, three machine B and two machine C
worked 120 hours. What is average of all 5A?
Q87)A CLAN WHO WRITE EVERYTHING IN BINARY. THE VILLAGE LEADER BORN ON 1101TH
DAY OF MARCH 1896. FIND THE NO OF DAYS AFTER WHICH HE HAS HIS BIRTHDAY FROM
THE START OF THIS YEAR.. OPTIONS WERE 72, 73, 111, 110
ANS IS 72.
72 .. as 73rd day is 13th of marcbhh 1985
Q88) the difference between the roots of the equation x^2+bx+C =0 is 2. Which of the
following is correct? (Options were given)

NMAT REPEATED QUESTIONS

And: b^2-4ac=4a^2
Q89)
#Nmat Ds - Another statement was given which had clear data .
Shyam has his home and school in same horizontal path if a library also lies in the same
horizontal path and distance of Library to school is 7 km and and library to home is 6 km
find the distance b/w Home and School
Ans: with this we will have two possibilities 1km or 13 km ,
Q90) A candidate add sum of 1st n natural nos but misses 2 consecutive nos and final result
comes as 816.
How many pairs of missing nos are possible? How to solve this?
22&23
Q91) A man had 2 cloths of length 3.5m and 8.75m. He wants to cut both the pieces in equal
lengths. How many min number of pieces can he cut it into?
Simple find the hcf and add the pieces.
And :0.35??? How??
Ans 7
LCM of 3.5m and 8.75m is 1.75m (length of equal pieces. (1.75*a=3.5 a=2,
1.75*b=8.75==>b=5).
Total No of pieces a+b=2+5=7
92. Three equations were given. Common roots between them were supposed to be found
x^2-5x-6 x^2-6x-5 x^2-9x+20 (maybe change in signs but these were the equations 5 was
common root
Ans: Only equations 3 have common roots
93. A principle of 10,000 is invested at simple interest @ 4% p.a. for 10 and 12 yrs. Find the
difference between them
Ans: 800
94. -30,-28,-26.
The sum of this AP is equal for two different cases. S(x) = S (y)
Find the terms x and y

NMAT REPEATED QUESTIONS

Ans:
95. No of two circles , B lies in circle A1 and P is outside circle B.
Q96) sum of all the numbers formed with 5,6,7,8
Q97)are points A,B,P,D concyclic?
Statement 1: BP is a tangent to circle A1. D lies on Circle B1 which intersects other circle.
Statement 2: A and D lie on the point of intersection of two circles . P is outside of circle A1
Q98) There are 100 bulbs manufactured by machine A and 400 by machine B out of which
200 are faulty. It is said that total of 50% bulbs are defective . A bulb is picked at random
and is found to be defective . what is the probability that it is from machine B
ans

getting ans as can somebody confirm ?

Q) shweta sells 2 jacketa A and B. She made a loss of 10% on jacket A. If she had sold it for
450 more, she wold have gained 25%. Cp of javket B was given. She earned some profit On
it by giving a discount. Find the ratio of the marked price of B and cost proce of A.
Q) A , B and C enter into a partnership . Their ratios we had to find out. Then total profit was
given. We had to find out the profits of A ,B and C. Then A and B enter into a partnership
with D with their profits. One condition was given to find the onvestment of D. Find the
ratio of invesyment of D and profit of C.
------------------------- LR ---------------------------1) 2,6,12,20,?,42
Find ?
Solution- 1^2+1=2,2^2+2=6 and so on 5^2+5=30
Ans. 30
2. Find wrong number in the series
210210, 14100, 1470, 210, 42, 14, 7
Ans- 14100 because 7*2, 14*3, 42*5,210*7 etc trend follows from right (multiplying with
prime nos)
7*2=14 , 14*3=42, 42*5=210, 210*7=1470, 1470*11 = 16170, 16170*13 = 210210
Decision Making
1. Due to the hot weather in the city, the birds and animals were dehydrated. Not exactly
this but something like this
Options were :

NMAT REPEATED QUESTIONS

A) The people of the city should keep water buckets outside their homes for the birds and
animals to drink.
B) the government should set up a water reserve or something outside the city. ANS - None?
Why not 1 ?
Symbols based relationship problems in LR part- 2 ques
2 Arrangement based problems- 1 Easy (Drawing competition- 5 girls- 5 different cartoon
characters-and their ranking), 1 medium difficult ( 4 teams-4 captains-2 different yearsRankings) DO YOU REMEMBER THE ARRANGEMENT SEQUENCE?
Arrangement based problem- 5 people wearing different coloured shirts and having
different pocket moneys.(Easy- Mod?)
Another Arrangement based problem- 8(4boys P,Q,R,S and 4girls A,B,C,D) friends sitting
around a rectangular table. 3 each on the longer sides and 1 each on the shorter side. None
of the boys sat on the shorter sides. None from A and B sat on the shorter side. P was to the
left of C. A was seated in between R and Q. C was to the left of Q. B was seated in between S
and P. (I have just changed the names since i dont remember the names, the question
was very much similar to this)
Sequence? The order in which the two caselets appeared? Yes
Easy one appeared first somewhere in the middle and the medium one was in absolute last

5-6 Course of action, conclusion, agreement problems


2 table series
1 question had symbols in a grid and a logical sequence was asked from the question
Machine input question (4)- deducing 1st step was difficult rest were quite easy-HOW WERE
YOU ABLE TO DO THOSE QUESTIONS? STUDIED FROM WHERE THE WAY TO ATTEMPT SUCH
QUESTIONS?
15th october window 2 LR was difficult, patterns were confusing & arrangement sets too
lengthy
Input
Step
Step
Step 3 d5

1
2

a
a1b4
c27

NMAT REPEATED QUESTIONS

Step 4 e66
Step 5
Step 6
Also I got an lr set with 5 people managing 5 stocks mid cap, blue cap, power, infra and one
more and rates of return were also needed to be arranged. Easy set must attempt.
Easy table di set :
It
was
something
using
this
Total flux in population = natural increase + immigrant emigrant

basic

concept

NMAT REPEATED QUESTIONS

Quant NMAT Window 4

Q2) ??
Ans 4C2 0r 6
Formula is n-1 C r-1 if condition is atleast one else n+r-1 C r-1
there are 10 billion people in the world today and 5 million square km of land. if each
person need 1 square meter of land then in which year will there be no space left if the
population doubles itself every year and we are in 2006. provide sol
Ans..2015

6-10 7-20 8-40 9-80 10-160 11-320 12- 640 13 -1380 14- 2760 15- 5520 billion
If it getting double every year It should go like 10, 20, 40, 80, 160, 2560, 5120 in 2015. So
ans is 2014 or 2015?? Cz in 2015 its beyond 5000{after simplifying billion n million}.
..Whats the final answer? 2015 is the final answer.
Q5) (10^0+10^1+10^2+...) mod 8
Ans 8 , 1+10+100+1000 .- mod 8
i.e 1+2+4+1^0 = 8 Beyond 100 each term is divisible by 8 (concept prime factor more than
2^3)
sol

anyone online ? if yes , we can mutualy solve our doubts . intertesd type yes

Shouldnt the answer be 7?


Answer should be 7 then !!

NMAT REPEATED QUESTIONS

here :
yes Ans should be 7
Q6) The CP of 3 kinds of sugar are 5, 6, 6.80; In what optimum proportion should they be
mixed so that the price of the mixture be 6.5 per bag? Ans.1:3:10

Solution?

5p
+
6q
+
6.8r=
6.5p
+
1.5p
+
.5q
now u can substitute and check-----Not getting this

6.5q
=

6.5r
.3r

Q8) 1^2-3^2+5^2-7^2+............- 99^2+101^2( please someone give correct qstn n


solution)ANS 5201
A) -5202 [can some one explain this please?] ( I think 5201 is the write answer ) y
Soln - .
= 1^2 + { Sum ( 4n+1)^2 - Sum ( 4n-1)^2 } ; where n = 1 to 25
= 1 + Sum ( 16n) = 1 + 16n*(n+1) /2 = 1 + 8*25*26 = 1 + 5200 = 5201
My answer too is 5201
What is the final answer 5202 or 5201?????
5201 seems correct
5201 is correct because addition of odd nos.(i.e.51 nos.) will always give you an odd units
digit
any other way
Q9) A mixture of 50 ml has ethanol and water with water being 30%. if 10ml of the mixture
is replace with water. Finding percentage of ethanol in the resultant mixture.
Ans) Its 56
Q) A, B, C can do a work in 10, 20, 30 days. Find from the below
options which had made the maximum amount of work. (5 combinations

NMAT REPEATED QUESTIONS

were given)
Q10.) a+1/a=5 find a-1/a; a is an integer
A) 21 (correct:verified)
METHOD PlS

By substituting 21 we will not get a as an integer. Any other answer or solution?


p Q11) Raj writes numbers from 1 to 200 on a sheet of paper how many are not divisible
either by 3 or 7 or both.
A)115(confirm?). I think it should be 114.
200-(66+28)+9 = 115 (y add 9?)
N(1- 1/a)(1-1/b) a=3 b=7 N=200 ans = 114

Q14) A dice is thrown probability that you get an even number after even number of
throws.
A): ? (even according to me, it should be 1/3)??????
final ans ??? (Answer is 1/3 confirmed!!!!) --- Can you expalin how? solution please?
Explaination: It is talking about even number of tries. Probability for both even and odd
numbers to be appearing is . So, the series will be like this: .+...+.......
Here a= and d= and it is an infinite series. So solution is: 1/4/(1-)=.

Q15) Probability of success of an event is 1 in 10. What is the probability of 4 successes in 5


attempts?
5c4(1/10)^4(9/10)
Q) Number of times digit 5 comes from 1 to 1000

NMAT REPEATED QUESTIONS

299 . 299 OR 300???? 300.answer will be 300..use formula..n*10^n-1...here n =3..for


range 000 - 999

Q16) The average of bowler is calculated by dividing the number of runs he gave by the
wickets he took. The average of P and Q were 4 before a match. After the match in which P
took 3 wickets for 24 runs and Q took 2 wickets for 26 runs, their average still remain equal.
Before the match, they had taken a total of 30 wickets between them. How many wickets
had P taken before the match?
A) ( 11, Solution below )

Runs and wickets of P before match = a , b


Runs and wickets of Q before match = c , d
0a/b=c/d=4
a=4b , c=4d
(a+24)/(b+3) = (c+26)/(d+2)
Also , b+d = 30
Solving , d= 19 , b = 11

Q17) How many ways can you arrange a group of 4, 3, 2 in a row such that all people (of a
group) stay together.
A.

4084(please verify answer, solution pls!!!

(***I think it should be 1728***) solution please,3,3 th


confirm ans pl I am getting 1728 = 3!* 4! * 3!* 2!

NMAT REPEATED QUESTIONS

If
it
is
a
group
of
4en
if it is a group of 4,3,2 then 4!*3!*2!*3! = 1728

it

is

4!*3!*3!*3!

5184

seems to be 1728 for me. I have not got 4048

Q18) Sn = 1+1/7+1/7^2+1/7^3 .... 1/7^n-1) Find value of n will satisfy 7/5 - Sn <=
1/100
A) sn=1+1/7 =8/7, 7/5-8/7<=100 , so answer will be n=3.,
plz explain if any other approach.

(7- 1/7^n-1) /6 = sn
Q19). If a person takes 10000 at 8% interest rate for 3 years (SI) and lends the same at 7%
compounded annually for 3 years what's his profit/loss %.
A.

1.2%loss

Q20). If A= 1+ 1/2^2+1/3^2+1/4^2....
B= 1+1/3^2+1/5^2+1/7^2 ....
What's the relation between A and by
A) 4B=3A(how?)
(On solving, we get A-B= A/4)
1solution is right we get A-B=A(+1/16+1/36)----->A-B=.25A -------- How?

Q21) Find the amount to be paid on 4000 if successive discounts of 30% and 10% Ans. 2520

NMAT REPEATED QUESTIONS

Find the sum of the digits of the least number n, such that 2n is a square and 3n is a cube
ANS: 2^3 * 3^2 = 72 therefore 9

For how many values of a, is the number 15^a+3 * 28^8-a / 35^2a-9 an integer
ANS: 8?
6 for a>=0 and 3 for a<0
(3^(a+3)*5^(12-a)*4^(8-a)*7^(17-3a)) I think its 9, is it?yeah 9*-3to 5]
Q 22) center o and AB chord and another point on circle in minor arc D...find angle ADB
i.angle AOB=50
ii. angle OAB=45
A 23) either explain if angle aob is 50 than using the angle of major arc will be 310..so using
that we can find angle ADB...if another
angle is given , we can find angle 7uAOB using sum of angle in triangle as OAB = OBA, so
using either 1 or 2

we can find the solution.

Q 26)what is x1` ?
i. Q )log(x+2)-log(6)=2 , find x...base of all log is x only..

NMAT REPEATED QUESTIONS

Ans.x=2/3
x is prime no.
ii. 51<= x <= 57
A)i

Q27 )
A)331

Q29) find A+B+C


data given in table;
20 40K A 0 160
30 B0 27 0 810
40 160 640 C560
A.

A=8,B=9,C=2

NMAT Window 3

One di from official mock repeated -green house emission in country,repeated on


12th,11.30(values were diff, 350 was total emmision, and other % were 20,30,15,23 i think)
Same values

Official mock means which mock? - NMAT GMAC free mock

NMAT REPEATED QUESTIONS

Q1. The difference of CI and SI of a particular amount kept for 3 yrs is 310. Find the principal
amt. R=10%
Options:
9000
10000
11000
12000
Ans-10000 repeated 7th
How to approach for this question?
>> P(1.1)^3 - P - 0.3P = 310
Q) wrong number in the series
14100, 1470, 210, 42, 14, 7
Answer : 14100?? Yes, rest are multiples of 7.
Q2. Plane hat time plane will reach the destination if distance is 1500km. schedule to depart
at 9 am.delayed by 30 mins.Driver increased the speed by 250kmph and reached on time.At
w
Let actual time taken be t
And the extra speed is 250 kmph
Therefore
[1500/(t-0.5)] - [1500/(t)] = 250

Solve the Quadratic Equation.

2(t^2) - t - 6 = 0
0Gives you t =1.5
Hence time of arrival is 11 a.m (repeated on 9th 2pm)

NMAT REPEATED QUESTIONS

If answer is 1.5 shouldn't the final answer be 10:30 and not 11 am ? (i am also getting
answer as 1.5). The departure was at 9.30 and not at 9. Hence it will be 11. -- Thanks a ton!
:)

Q3 train is 20 faster than car.train and car starts from a point and reaches at the same
time to a point 75km away.Train halted for 12.5 mins at stations in between.if train is 25
faster then how much time will it take to reach the
Speed of Car = 60 kmph
Given if train was 25%faster ,then Speed of train = 60X1.25= 75kmph
Then time taken = Distance/Speed of train = 1 hr (Should this time not include tsame point.t
repeat on 9th repeated on 9th 2pm
Ans : 1 hour How? Please explain?
Let Ratio of speed of car:train = 5:6
Since distance travelled is same therefore by equn:
5X(Tcar)= 6X(Tcar-12.5)
Hence Tcar =75min Now, Speed of Car = dis/Tcar = (75X60)/75
hour)he halt time? I think it should)

(conversion of minutes to

(It will include the time of halt so the answer will be 1 hour 12.5 mins)
Q4.Ans. 433
What about 135??433 it is.. (greatest 3 digit number less than 500 is
asked)1+3^2+5^3 (appeared on 12th)
How ?? please explain..
Take a=9 , b=9 , c=7
C cannot be 8 so take it as 7 =7^3= 343
B max can be 9=9^2=81
A max can be 9
Hence, 343+81+9=433<500.
Approach?
Please post question?
What is the condition of the question??
What is the question?

NMAT REPEATED QUESTIONS

Q.5 How many ordered pairs (a,b,c) exist such that their LCM is 400 (appeared on 12th)
Ans. LCM (a,b) = 400 = 16*25
Let us think from the perspective of each prime.
First of all, both a and b must be factors of 24
So we have 5 possibilities of both a and b. Hence 52
possibilities.
But from these possibilities we must remove those cases where a and b are neither 24
. This will happen when both a and b are factors of 23
So that is 42
cases that must be removed.
So, we have (5242)
cases where the LCM will be 24
Similarly, for 52
, we will have (3222) cases where the LCM of a and b will be 52
So, we have a total of
(5242)(3222)=45
cases where LCM (a,b) = 400
These are the/ ordered solutionso

Ans may be is 91 pls tell the options too??


Can anyone explain the above solution? How 400 = 24*52 ?? What is the final ans?
Any different Approach to solve dis?
If N= a^m*b^n *c^o then ordered solution such that LCM = N is (2m+1)*(2n+1)*(2o+1)
Unordered will be { (2m+1)*(2n+1)*(2o+1) /2} +0.5

NMAT REPEATED QUESTIONS

400= 2^4 *5^2


Therefore, ordered pairs = (2*4+1)(2*2+1) = 45
1159 is d ans shyd
Yes 1159 is correct can you explain how 1159?? Also, if you remember the options plz
tell!!
Q6. How many 6 distinct letter word can be formed from the set S having 3 subsets, (Y,u,H),
(T,a) and (I,c)
Ans: 7! (7 letters can be arranged in 7! ways)
How?
First letters are selected as 7C6 and then arranged as 6!,hence 7C6*6! = 7!
Q7. The 1st, 5th and 11th term of an arithmetic progression series are in geometric
progression. The sum of 6th and 9th term of that arithmetic progression series is 63. What is
the 21st term of that arithmetic series..? Repeated from window 2
Whats the final answer ?
Ans - 28* 63 / 29
T5^2=T1*T11
(a+4d)^2=a(a+10d)
On solving this we get a=8d
Now t6+t9=63
Put the above value we get d=63/29 and for 21st term i.e a+20d=28*63/29
Alt. solution
Let first term be a and commo uhn diff d
T6 + t9 = 63
(a+5d) + (a+8d) = 63
2A+13d = 63
Only (a,d) pairs satisfying are (25,1) and (12,3)
And (12,3) satisfies g.m. condition so t21 becomes 12+ (3*20) = 72 -> 12,3 doesn't satisfy gm
condition. The first approach is correct
But with the first approach the answer that is coming is correct (as per method) then how
the second method shows different answer please someone clearify How does (12,3)

NMAT REPEATED QUESTIONS

satisfy the G.M condition ? The 1st term will be 12, the 5th term will be 24 and the 11th
term will be 42 which does not satisfy G.M. condition.
Anyone Plz explain? Also what were the options like? Were they fractions too?
Q8. There are 3 drawers and 5 different coloured pens/balls. In how many ways can you put
the 5 pens/ balls in the 3 drawers so that there is at least 1 ball in each drawer?repeated 7
on 8th,12th (options were 150,60,90,45,and one more i dont remember)
Ans 6?
Dont get confuse..150 is the right answer..mark it only
Possible ways : (2 2 1) & (3 1 1) .
(2 2 1) - 5c1 * 3 =15
(3 1 1) - 5c3 * 1 = 10. So total 25. Assuming drawers to be similar. Can someone explain this?
I am getting 150 as the answer! (IF DRAWERS ARE SIMILAR..THE ANSWER SHOULD IGNORE
ORDER OF SELECTION .i.e. 150/3! = 25)
311 case- 5C3*2C1*1C1*3 ( as 311,131, 113)= 60
221 case- 5c2*3c2*1c1*3 (as 221,122, 212)= 90
Hence total= 150(this is correct)(100%)
3^5-(3c1 2^5)+3c2?
6 will be the answer if similar balls . some1 pls explain distinct
It should be 5C3 *3^2 in case drawers are identical
>>>Possible solution: (Dont know if it is correctThere will be two cases : 311 or 221
So Ans = [(5c3*3c1)(2c1*2c1)(1)]/2 + [(5c2*3c1)(3c2*2c1)(1)]/2 =60 + 30 = 90
Approach??
All the options were double digit.
Number of ways of selecting three distinct balls from 5 is 5C3 and number of ways of placing
the remaining 2 distinct balls is 9. Hence 5C3*9=10*9=90(NOT SURE THOUGH)(???)
What is the final answer for this question ???? 9 60/90/150/25 ????

NMAT REPEATED QUESTIONS

->from 5 balls first three balls are selected and arranged 3 drawers in 3!ways.next remaining
2 balls are arranged ...by selecting one drawer from 3 and placing in it or by selecting 2 and
placing in them in 2! .it is done as
=>5c3*(3c1+3c2*2!)=60
150 ans h shyd ...
Q13. What is the sum of the series till the first 21 terms, in which nth term is given by
3+5^n? repeated 7t
Ans : 63 + 5(5^21 - 1) / 4 ...this is correct!

(how?)

Q9. A,B,C pipes are used to fill the tank. A and C takes the same time as B alone takes to fill
the tank. C is 3 hours faster than A and 1 hr slower than B.How much time will A alone take
to fill the tank.repeated 7th and 9th
Ans -the answer is 6
Let the time taken by B alone = x
Hence, time taken by C alone = x+1
And time taken by A alone = x+4
Now, from the question we get,
1/(x+1) + 1/(x+4) = 1/x
Solving the equation you will get x= 2
Therefore, for A it will be 6 hours.

Q10. On selling something, there is a profit of 12 %. If the CP is decreased by 4% and SP is 10


more than original SP,20% profit is achieved.What is the original CP? Ans ? repeated on
8th,12th 11.30 Ans: 312.5?
approach?
cp=x sp=1.12x0
cp=0.96x sp=1.12x+10
[(1.12x+10)-0.96x]/0.96x = 20/100

NMAT REPEATED QUESTIONS

Solve for x.
Q11. What are the coordinates of the incentre of the triangle formed by the coordinates
(3,1), (10,1) and (3,25) (repeated 12th)
Ans- (6,4)
Solution: The triangle formed is a right angled triangle with sides 7, 24, 25. ??
Incenter=
{ (ax1+bx2+cx3)/(a+b+c), (ay1+by2+cy3)/(a+b+c) } a,b,c sides of triangle =7,24,25
Q12. Find the TSA of a hemispherical bowl with external radius R and uniform width of R/8?
Repeat on 9th Nov,12th
Ans: i am getting : pi*R^2*(241/64)
TSA of Hemispherical Shell= 3*pie*R^2 + pie*r^2
Here r=R-R/8 = 7R/8
Substitute in formula.
TSA is always inner plus outer and for the top we consider the width, hence u can
understand the formula as 2pieR^2 + 2pier^2 + (pieR^2 - pier^2)= 3pieR^2 + pier^2 hence
the formula is correct!!
Shouldnt it be 2*pi*R^2 - 2*pi*(7R/8)^2 + pi*(R/8)^2 ?--> WRONG
Q14. What is the value of (n+1)^3 + 3(n+1)^2 + 3(n+1) ? repeated 7th
Ans- n^3
+ 6n^2 + 12n + 7 (just use the identities and expand)
Can some one explain the question n ans here??
Put N=1 and eliminate ans choices (best approach )
Q15)Find the difference between si and ci if p=8000,r=20%,t=3years
Ans-1024
Pr^3+3Pr^2=Pr^2(r+3)
8000*(0.04)(3.2)=320*3.2
=1024 repeated on 8th,12th
When difference b/w SI and CI is asked use the following formulae:.

NMAT REPEATED QUESTIONS

When no. of years is 2,


(D*100^3)/R^2*(300+R)

P=(D*100^2)/When no. of years is 3,

P=

D=Difference

Q16. A conference has to be attended by 10 ministers of different states, in how many ways
can you arrange them on a circular table so that the ministers from kerala, Karnataka and
Tamil Nadu always sit together..(repeated 12th)
Options: 5040, 20160, 30240, 40320
Answer is 30240
Consider the 3 as 1 unit. So we are left with 7+1 i.e. 8. They can be arranged in (8-1)! Ways,
i.e. 7!. Now the 3 can be arranged in 3! Ways. Therefor total no. of ways = 7!*3!=30240

In a circular table, 10 ministers will be arranged in 9! Ways.


So then, other 3 being considered as 1 unit shouldn't it be 6!3!?
Please verify someone.
If we consider other 3 as one unit then,
Total units = 10-3+1=8
Therefore total arrangements 7!*3!

Guys one small doubt here?


like necklace arrangmnt wont we consider same thing here as well?that is 7!3!/2 clockwise
or anticlockwise?

Q17. A Cone is cut out from a Cube. Total surface area of cube is given. Find volume of Cube
left after cone is removed repeated on 8th and 9th

NMAT REPEATED QUESTIONS

(Cone of max volume? think so..


=>height of cone = diameter of cone = side of cube)->correct
Volume of cube - volume of cone = remaining volume ???? confirm?
Is it x^3 - pi x^3/12 ?? Yes!

Q18. Last digit of 1!+2!+3!.....100!


Ans: 3
1+2+6+4 = 3

dCan anyone explain the approach??


1! Last digit is 1, similarly for 2!,3!,4! Will be 2,6,4 resp. after that when 5! Will come it will
always have a 0 at the end so last digit will be decided by 1!,2!,3!,4! Only.

Q19. Car moves from a to b..other from b to a..time was given.. at what time they will cross
each other.
Ans: relative speed concept invloved.
Meeting time: t=d/(S1+s2)

Q20: 1 car leaves from pune at 10 am and reaches bangalore at 8pm. Another car leaves
bangalore at 10am and reaches pune at 10pm, what time do they cross each
other?repeated 7th
Approach? Ans ?

NMAT REPEATED QUESTIONS

Let the distance be X


Speed of Car 1 is X/10
Speed of Car 2 is X/12
X/(X/10+X/12) and hence the ans would be 120/22 = 3:27 pm
5+0.45*60 = 5:27 hours therefore 10 +5.27= 3:27PM

Q21: data sufficiency: can you find m/n


S1: m-n =4
S2: mn= 96
Both the Statements are needed
I think even after using both the statements it cant be found out because values of m and n
vary n hence their ratio also chngs. Which other values? Cause i got only 12,8. -12 & -8 can
also be the values. Yes using both also we cant get ans.
repeated on 8th
Cannot be determined even with both the Statements
We can find the value of n which will be 12 and 8. For these two values of n we have two
values of m. So now m/n will also have two different values. So we cannot reach a unique
solution and therefore it is cannot be determined.
Q22: a fills tank in 10 mins, b empties in 6 mins, how much will it take to empty of the
tank if both are open.repeated 7th,8th
Let the Tank Size Be X
In 1 Minute A fills X/10 tank
In 1 minute B empties X/6 Tank
So in 1 minute a net of X/6 - X/10 tank empties
In 1 minute x/15 tank empties so to empty it 3X/5 tank we will need
15*3/5 =9 minutes
Ans : 9 mins. correct

NMAT REPEATED QUESTIONS

23) Find the probability of getting prime number sum when 6 dices are rolled?
I think: No. from 6 to 36 can appear on the dice, which is 31 numbers. And prime no. sum
can be only 7, 11, 13, 17, 19, 23, 29, 31. So, ans. should be 8/31. Please correct me if I am
wrong?
APPROACH PLEASE! 1/4 repeated on 8th
What is the correct answer?
Whats the correct answer? Coz 7 can appear in 7! Ways (1111112)(1111121). So on..
We need a prime number sum from Identical dices
A+b+c+d+e+f = 7
A+b+c+d+e+f = 11
A+b+c+d+e+f = 13
A+b+c+d+e+f = 17.
A+b+c+d+e+f = 31
Now using n-1Cr-1
What is the correct answer? Ans ???
1) 2,6,12,20,?,42 6-2=4
12-6=6
20-12=8
therefore , 30-20=10 correct! 30 is the answer
Find ? 30 (Approach is n^2 +n)
(cant understand the question)??
24)Number of ways you can represent 180 as a sum of consecutive numbers method is
5(number of odd factors of 180). Since page number on each leaf is two times so number of
consecutive numbers has to be even start with 4 because we know it cannot be 2. It should
be 4
AKSHITA CAN SOMEONE MAKE SENSE OF THE 24th QUESTION???(The no of consecutive
terms will be 5-(3,5,9,15,45),because the no of odd factors of 180 is 6,
But 1 is not considered in this case,as we need consecutive terms sum).

NMAT REPEATED QUESTIONS

For eg.
100=2^2*5^2
So Number of odd factors = (2+1) - 1 = 2 = Number of ways of writing 100 as sum of 2 or
more consecutive integers . They are
18, 19, 20, 21, 22
9,10,11,12,13,14,15,16
Similarly for 180=2^2*3^2*5^1
So , no of odd factors here are = (2+1)(1+1)-1=5 [consider factors for 3 and 5], which is the
required answer.
24).(repeated on 8th)There was a question, something like this: there are three types of
alcoholic mixtures with 80% 70% and 50% concentration of wine. In what ratio should they
be mixed to get a 60% concentration??
What is the answer for 24???how to solve this??
Method plz?
Ans is 1:1:3- correct
It is infact 1:2:2-wrong
25) Trailing zeroes in 60! When expressed to the base 12. (9th nov) Ans 28 repeated on 9th
2pm shouldnt mthe answer be 18? Highest power of 12 in 60! Is 18 therefore 18 trailing
zeroes Method please?
Confirm? Its base 12 thats why answer will be 28 and not 18
26) Trailing zeroes in 100! When expressed to the base 7.(9th nov) Ans
16? Even i got 16? Someone confirm pls.Yes, it is correct! Number of trailing zeroes will be
the highest power of 7 in 100! Which is 16.
Approach ?
100/7=14 then 14/7=2 further the quotient cannot be divided so ans is 14+2=16
+

NMAT REPEATED QUESTIONS

27) product of sum of 2nos and their hcf is 36. How many such pair exist? (9th Nov) = 10
(1,35)(5,31)(7,29)(11,25)(13,23)(17,19)(2,16)(4,14)(8,10)(3,9)
Last 3 sets r wrong as they need to be coprime instead the sets will be (1,8) (2,7) (5,4) (1,3)
Total 10
The correct answer is 44. The question says product of ( sum of 2 no.s and their HCF)
repeated on 9th 2pm
28) DI caselets total 6 (9th nov) what is this?
29)probability of wining a ballot of 3 states A,B,C was given. Total amount of votes in a,b,c
were given. Question asked was prob. That it is a win and belong to state a.(8th)
30)
One ds question:-(8th)
Find the value of x,y if some equation was given
a) This equation was reducible to question equation itself.
b) Another equation was given.
So the answer was only 2nd is required.
31)find the remainder if 8533^859^361 divided by 19?(8th mrng) Ans 3?: 3 is coming
Is the ans 16? Approach
32)how many 7 digits distinct number can be made with 1,2,3,4,5,2,1 if on odd places only
odd numbers can be used?(8th)
a)12
b)24
c)36
d)48
e) None of these
I guess yehi options the
Ans 36? Yes .Even i got 36, but considering that repetitions not allowed. Is that right?
HOW COME 36? - NUMBERS HAVE TO BE DISTINCT BUT YOU HAVE TO USE ALL THE 7
DIGITS. SHOULDNT IT BE - ( 3!/2! )*(4!/2!) = 18 ? -> this is 36 only. Plz solve again :)

NMAT REPEATED QUESTIONS

What about logical reasoning?


Please post some questions of LR also, If anyone remembers??
There was series questions
32,34,37,41,47,46,58,56,? (9th nov)
What is the ans?60-ans
Approach?
How 60?
Was 80 an option???
Should be 71 correct answer
32+(3+2)=37
37+(3+7)=47
47+(4+7)=58
58+(5+8)=71
Same goes for the other enven term series 34,41.
I dont think so that the above question should be that complicated(for ans. 71) . The
correct ans should be 80. Was 80 an option?
11th Nov
1. A train starts frm Pune from 10 am and reaches bangalore at 8 pm. Another train starts
from Bangalore from 10 am and reaches Pune at 10 PM. At what time will they meet ? (ans
3:27 pm)
2. A pipe fills a tank in 12hrs another pipe can empty it in 6hrs. If a tank is 3/5 full, in how
much time will it be empty. 7.2 hrs
3. a group of 5 men have average weight of 142 pounds. Another group of 2 women having
total weight 200 pounds is added in first group. WHat is the average weight of complete
group ?
Is the answer 130 ?
Approach - (Sum of wt of 5 ppl)/5=142...eqn 1
(Sum of wt of 5 ppl + 200)/7...eqn 2

NMAT REPEATED QUESTIONS

Subst. Sum of wt of 5 ppl as 5x142 = 710.


Hence, (710+200)/7 = 130
4. A dice is rolled 2 times. what is the probability of getting sum atleast 3 and atmost 5 ? Is
the answer 9/36?= ?100% correct 1/4
5. Is 3^p < 2 ^q ?
I: q=2p
II: q>0
(Using both statements?) yes
6. There are 6 pink balls, 6 black balls and 7 white balls in a bag. 3 balls are drawn at
random. WHat is the probability of getting exactly 2 balls of same colour ? (214/323?)i got
221/323 --REPEATED ON 12th Morning
214/323 is correct..approch (6C2*13C1+ 6C2*13C1+ 7C2*12C1)/ 19C3 = 214/323 ...thnxx a
lot.
7. A girl has 5 pens of different colours and she has to put them in 3 drawers. Each drawer
should have atleast 1 pen. In how many ways she can put all the 5 pens in the drawers ? 150
8. In a meeting of 10 states, representatives are sitting in a round table. In how many ways
can the representatives of Goa, UP and MP are sitting together ? Ans: 7!*3!
9. A red tap can fill an empty tank in 40 hrs and green tap can fill it in 30 hrs and blue tap
can empty the same tank in 25 hrs when operated individually. How much time would it
take to fill the tank when they are operated alternatively every hour? 162 hours 24 mins ??
(repeated 12th)
Yes, Im getting the same answer. Looks fine !
Method plz??
10. In an isoceles traingle AC=BC. AD is angle bisector of ang(BAC).What s the ratio of
Incenter and Circumcenter. 1: sqrt2
Can anyone explain the answer??

NMAT REPEATED QUESTIONS

11. In an isoceles triangle ABC, external ang(ACD)=100 degree. What is the max possible
angle in the triangle? (80) repeated 12TH
12. 3 co ordinates were given and we need to find out the cordinates of incenter.
13. 3x^2-10x+b=0. The roots are reciprocal. Find the value of roots and b.(3 & and b=3)
plzz share approch (12th)
14. 7, _ , 33, 52, 75, 102 (Came on 10th and repeated on 11th) (Ans:18?) YES 18 SHOULD BE
CORRECT, 7, 7+11, 18+15, 33+19, 52+23, 75+27
15. 1,2,3,4,5. How many 5 digit numbers are there which are divisible
by 4.(24?) is 24 correct ans..??
16. Difference between SI & CI for a principal for 3 years with r=10% is 230. Find the
principal.
17. Sam has taken an amount of 8000rs on r=20% pa for 3 years on SI. But later Harry
insisted Sam to pay it compoundedly. How much is difference in the interest? (ans: 1024?)
18. On selling something there is P=12%. If the CP is decreased by 4% and SP is 10 more than
original SP. 20% profit is achieved. Original CP?
(312.5).
19. 3 persons A, B , C with speeds 4 kmph 6kmph 7 kmph respectively
start from a same point P after evry 1 hour in the same order. As soon as B meets A at point
Q, B returns back to point P. Where will B & C meet? (9.69km from P)(12th)
20. Discount of 20% is given on CP but seller cheats by 200gm per 1000gm while selling.
What is the profit/loss% ? No profit no loss ->
5 I am getting 100% Profit. Method isLet cost of 100 gm is 100/But Seller buys 100gm at 80/So he must have bought 200 gm for 160/-

NMAT REPEATED QUESTIONS

Now he sells 100gm at the cost of 200gm as he is cheating for every 100 gm
Means he actually sells 100 gm for 160/- by saying it is 200gm
So 200 will be sold at 320/Profit %= sp-cp/cp
320 (for 200gm)- 160 (for 200 gm )/ 160= 160/160= 100%
Guys, plz confirm if this is correct
cp-100 dis-20% sp-80
and while cheating he used to give 20%less
in quantity of 100 he gives 80
therefore 80 quantity at 80rs . no profit no loss

Was 74 an option?can someone provide the solution for this?


12TH Questions:
1. What is the highest power of 11 that will leave no remainder on dividing 4000!?
(Ans: 399?)
2. A and B swim downstream from X to Y. A makes 60 strokes to cover the distance and
B makes 90 strokes. If the length of the strokes is same and A can make double the
strokes than what B can make then how many strokes will a have to make to cover
the distance XY in still water? (I am not 100% sure if all data is correct but this was
the outline of the question).
3. X,Y,Z can do a work in 10 days working together, if X takes double the time of what Y
and Z take working together, than in what time can X alone complete the work?
any1 with the solution plz
ans. 30
4. If a sum doubles itself compounded biannually at r% rate of interest in 123.6/r years,
then what will be the amount if sum of 2400 is invested at 12% rate for 20 years?
(options were : 4800, 6000, etc) Ans?
5. Very long partnership question involving 3 people and different times of investment
and 2 time investing.
6. Data sufficiency: Rectangle has to be covered by 2m long tiles. Can you determine
how many tiles are needed if :

NMAT REPEATED QUESTIONS

a. The semiperimeter of rectangle is 8 times the area of rectangle and the


length is 12 more than the width
b. Width is in some proportion of length.
Ans: Can be calculated with statement A only!
7.data sufficiency- Percentage increase in rectangle is?
a) Length increase by 10% and width reduce by 12%
b) Ratio was given, i cant remember the exact figures,
Ans- by each alone you can answer

14th and 15 th November Questions -

1- Product of sum of 2 nos and their hcf is 36 . How many such pair exists ?
Ans - 10.
2- 12 cubes were placed in an L shape . What is % loss in TSA ?
Ans - 41.67%

N INPUT & OUTPUT QUESTIONS (logic for below?? Pls share)


I/O: (11th Nov)

Input: 24 57 68 91

NMAT REPEATED QUESTIONS

StepI: 06 12 14 10
Step II: 18 45 54 81
Step III:
0000 09 09 10
Step IV: 28 54 63 91
Step V: 16 20 18 09

Q: Step V for 48?


Q: Step 2 for 18? Ans 9
Step1 sum of digits
Step2 - (Input) - (Step1)
Step 3 : (Can anyone please explain step 3?)????
Step 4 - Step 2 + Step 3
Step 5 - Multiplication of the digits of the number in step 4 (For eg in the case of 28, its 2*8
= 16)

Aman and Amani have 3 sons : Ren, Zen, Ben who married to each of 3 daughters of Imran
and Imrani: Rai, Bai, Sai. Sai and Bai are sister in laws of Ben. Zen was Bai's brother in law.
Which of the following are couples? [Ans: Zen:Sai and Ren and Bai)

Q: 5 children with different ages from : 4 , 5, 6, 9, 11 and they are brand ambassdors of 5
different things. Question on this.

NMAT REPEATED QUESTIONS

Q: A photograph has 5 people standing in a row. No 2 males or females are standing


together. the photo has a mother with her son and daughter who are with their spouses.
How is the person standing in the left od the middle person related to the person standing
in right of the middle person?
[Ans: Brother in Law]
Arrangement is Daughter|her husband|Mother|Son|his wife
Why not?
mother\son\his wife\daughters husband/daughter
SOLN: Still reln is brother in law itself.

Questions on weak argument, strong argument, causes of action, strengthen the argument,
weaken the argument etc

Q1
Input: Alphabets in pairs of 3. Ex: BCG
Step 1:237
In the table 237 is given
Step 2: 6 Logic:6 pls explain this ?
Logic: 2+3+7=12

3-2=1, 7-1=6

NMAT REPEATED QUESTIONS

For step 2 this is the logic.


Step 4: 24
Logic: Multiply step 3 digits with prime number
12x2=24
Next number in another column was multiplied with 3 and then with 5 and then with 7
Step 5: 22 (output)
Logic: Subtract that number which is multiplied in step 4Q

------>>> Four columns were there n 3-4 questions

And each asked for different steps


Like what will be the step 3 for input FAG

Ans.
Input : FAG
S1: 617
S2: 12
S3:14
Input output for 12th

QUANT NMAT WINDOW 2


Quant
Q.1 The fourth and the ninth terms of a harmonic progression are 1/10 and 1/25 then find
the series.
Ans: 1,,1/7,1/10
1. Yes this is correct series. Explanation: take the reciprocal of the terms given in the
question and solve accordingly.10 is the fourth term of AP, and 25 the ninth term.
SO, equating 10=a+3d and 25=a+8d, we get d to be 3.Thus, 3rd term of AP:7; 2nd: 4
and so on; after obtaining it, take its reciprocal again to get the ans. `

NMAT REPEATED QUESTIONS

Q
.2. 4 red, 5 blue,7 green balls hai. Two balls are drawn. Probability that they are of same
color ?
Ans: 37/120.

Total probability of drawing 2 balls= 16C2 =120. Now, cases of drawing similar color balls:
4C2+5C2+7C2= 6+10+21=37. THUS, 37/120. How are we using nCr here.The definition of nCr
says all n and all r should be distinct. But here all balls of the same colour are
indistinguishable.(Or are we assuming them to be distinct?
Q.3 If 21st of august falls on a thursday( don't remember the day exactly) in a leap year,
then 2nd of sept falls on what day after 3 years.
Ans: Friday. (Exactly next day)Repeated 2nd nov
SOLUTION? Can someone provide the approach for such questions?
Ans.
Count
the
no.
of
odd
days.
44Q.4. P is a no. which has exactly 3 factors which divides 25! Then whats is the greatest
power of 3 in (p-25)!
Ans: 46
SOLUTION :
Prime numbers less than 25 are 2,3,5,7,11,13,17,19,23 (unable to understand why this
step?)
When you find the highest power of each in 25!, we can see that highest power of 11 in 25!
is 2 and hence has 2+1=3 factors. Thus, p=11^2=121. Now find the highest power of 3 in
(121-25)!=96! and the ans=46.
Q.5. One third of one fifth of one fourth of a number is less than original number by 118.
What is the original number.
Ans. 120 Repeated 2nd nov
Explanation:let the no be x
So, (x-***x=118) = 59x/m60=118 Therefore x=2*60=120---- Repeated on 28th
Q.6. Salary of A is 30 less than that of B.So by what percentage Bs salary exceeds As?
Explanation: consider B salary to be 100 and A salary to be 70
B-A=100-70=30
Salary of B greater than A is given by: (30/70)*100=42.85%--- Repeated on 28th

NMAT REPEATED QUESTIONS

Got repeated today.


REPEATED 28TH OCT
Q.7 No of ways of choosing 5 dishes from 6 main course and 4 starters such that at least
one starter is always included.
Ans: 246 ways
Explanation :It is easy to find Total number of cases - No starter selected.
10c5 - 6c5 = 246 (works in all sums where at least one is asked)
Got repeated today.(27 Oct)

Q 8. Last digit of (3^155)*2


Ans- 4 Repeated 2nd nov
Please explain?
Explanation- 3^155 => Remainder[155/4]= 3 => 3^3 =27*2 => Last digit =4
Repeated 2day 27 oct--- Repeated on 28th

Q 9. Day of 26th Jan 1950 : Thursday ( repeated 27th oct)


-Was there any reference date and day given!? Not Required. Calculate from Jan 1 1900.
Split 1950 in centuries divisible by 400, i.e. 1600+300+49, 1600 would have 0 extra
days(why?), 300 will have 1 extra day and 49 years will have 49(minimum extra days)+
integral value of 49/4=12, so, 61 extra days. Total 1+61=62 extra days. And for months we
have 26 extra days. 62+26=88. 88/7 gives remainder= 4. (great explanation)
** Five DI sets : two caselets, one pie chart, one bar graph (easy), one mixture of line and
graph.
Q 10. In a triangle ABC, AD is median, M and N are mid points of AB and AC resp. Coordinates of A is (2,3) and M is (-3,5). Given that N lies on positive X axis, Find Equation of
Median. Length of MN is rt34.
Now it is complete
APPROACH PLEASE ???
Answer: IMO, the equation of AD is x-5y+13=0. So is this the correct answer(x-5y+13 = 0)?
Yes as As values satisfy
Correct ans is y=x+1
I got a different answer. My approach is as follows:

NMAT REPEATED QUESTIONS

Let C = (x,y). As MN = sqrt(34), BC = 2*sqrt (34). Now since N lies on X-axis => y-coordinate
of N is 0 => y coordinate of C = -3. Now since M(-3,5) is the midpoint of AB => B = (-8,7). If
we use distance formula for BC = 2*sqrt (34), we get x-coordinate of C as (-14) or (-2). Again
using mid-point formula, we get coordinate of D as (-11,2) or (-5,2).
Using two point method, as we have coordinates of A and D, we get equations as:
X-13y+37 = 0 or x-7y+19 = 0
Please confirm if any such option was there? Don't remember the options. Maybe it was
there. For saving time I just satisfied the given options with A(2,3
This also repeated today( 27Oct.)
Q.11 This question is incomplete. There was a diagram given for this question and ques was
to find the ratio of area of 2 triangles.
areas since the triangles are similar.
DEF:EBA area will be 16:49 (squares)
Now, we can apply the trapezium formula, under root of a*b, we get DEB and EFA will be
28cm square.
Therefore, ADF/EDF = 28 +16/16 = 44/16 = 11/4
Let me know if it is still unclear, will elaborate.
Please elaborate . Ok Which part is unclear? Trapezium formula? And how did you get deb
and EFA as 28?
You simply need to find the ratio of AD / ED. Now AD / ED = (AE / ED) + 1. Also, triangles DEF
and AEB are similar. So, ED / AE = DF / AB = 4/7. Put that above. You will get AD / ED as
11/4, which is the ratio of the required areas.

Q 12. Minute hand overlaps hour hand in 65 mins. In how many days it will gain 1440 hours
?
(faulty clock concept). 27th October repeat
I didn't understand the question!
Minute hand gains over hour hand means minute hand covers 65 minutes more every day??
-> Question rectified. Kindly check. SOMEBODY PLEASE SHARE APPROACH
If minute hand overlaps in 65 min, its a fast clock.
Therefore, it gains [(720/11) - 65] / 65 = 1/(11 x 13) minutes in 1 minute

NMAT REPEATED QUESTIONS

Thus, it gains (60 x 24)/(11 x 13) in a day.


Therefore, it gains 1440 hours in (1440 x 13 x 11) / (60 x 24)
Ans 143 (is this correct?)
Yes, I also got 143
Q15. In a party, there are rows arranged with 12,20 and 30 chairs and there were 3 left in all
the cases. When there were 11 rows. There were no chair left. If 30 chairs were added. Then
what will be the reminder?
Total chairs will be 363.(how?) Adding 30 we get 393. Now remainder has to be found for
how many rows?
Please explain how 363 is calculated ?
( since when divided by 12,20 and 30 we get 3 as remainder . So, the no. is of the form
180x+3, where 180 is lcm of 12,20,30. Now 180*2+3 is divisible by 11 so we get 363) The lcm
of 12, 20, and 30 is 60 and not 180, yes but 363 is right.

Total chairs come out to be 363. Adding 30 we get 393. Now since 11 rows are there, each
row will have 363/11 = 33 chairs. Now when 30 chairs are added, remainder will be 33-30=3.
But not entirely sure.
Can someone confirm?
I think we cannot assume things till we know for how many rows we have to find the
remainders ? Help( plz? 8 will be the answer for this as 30/11 remainder is 8
Yes 8 should be the answer. 8 is the correct ans. Do look for number of rows asked in
question 1st..don't blindly mark answer.
363 how?
a)Least number which when divided by a,b,c and leaves a remainder R in each case = ( l.c.m.
of a,b,c) + R
b) Now LCM of 12,20,30 is 60 and R = 3
c) Hence number is of the form 60x+3
d) Now 60x+3 is divisible by 11 since no chairs were left.
e) 363 is the smallest number which satisfies above condition.
But whats the point doing all these things , simply divide 30 by 11 and remainder is 8
SPOT ON
On 4TH nov, question modified to if we form 14 rows, then total how many chairs will be
left out? So 39314 remainder=1 ans.
===========================================================================
==km

NMAT REPEATED QUESTIONS

Q16. Find the difference between sum of all even numbers from 1-400 and sum of all odd
numbers between 51-180 ?
Ans. 32776
(Repeated on 27th : Different format (asked for sum of all even number till a
certain number like 3200: Need to find answer( not as given below)--Repeated on 28th-Suppose n is even and m is odd,with m>n, find sum of even from 1-n.? 28th repeated(Was
32776 in the options)
This is the only correct answer, ignore all others ==>Sum of all even nos from 1 to 400 so
from 2 to 400(including 2 and 400 both) i.e. 200 nos whose sum would be 40200(n^2+n)
where n=200
Also odd numbers between 51 to 180 i.e 53 to 179 which are 64 numbers so sum would be
7424 (using n/2[2a+(n-1)d) (why 51 is not taken while ad)
So 40200-7424 gives 32776. T he sum would of 53 to 179 would be 7424 (using n/2[2a+(n1)d] where n=64, a=53 mj

I checked and re-checked the 2nd sum is coming out to be 5728 and not 7424 Anyone with
me?
===========================================================================
=======

Q 17.A and B together complete certain work in 12 hours and one of them can finish in 10
hours less than other.. Find the average of time taken
Options- 10 15 20 25 30
Answer - 25 ..approach?

Let days be x and 10+x. Now


1/12= 1/x + 1/ (x+10)
12=x(x+10)/(2x+10)
REPEATED 28TH OCT
On solving this Quad eq, we get x=20, x+10=30
Thus average becomes (20+30)/2 = 25
Use options
25*2=50
50=2x+10
x=20 and x=30
Put the values. You will get
25... repeated we question

NMAT REPEATED QUESTIONS

Got repeated on 28th.


===========================================================================
=======

Q.18 rectangle dimensions were given,2 doors dimensions given, 2 windows dimensions
given. 70cm wide wallpaper to be covered on the walls of this room. Cost of wallpaper is 60
per meter.. calculate total cost
Repeated on 27ther:
The approach will be
area of rectangle- area of 2 doors - area of 2 Windows= The area left which would be
covered with wallpaper
Calculate cost per cm and then multiply with area left.
Dont get the solution.. where to use the 70cm length??
To calculate cost per cm as the cost is given per sq meter-repeated 27 Oct 9am how to
calculate cost per cm?
Can someone pls elaborate the question
REPEATED 28TH OCT-****************
===========================================================================
=======
Q.19 Shaded region question with a circular shaded park inside a square. Circular shaded
track inscribed in the square. We need to find the area of non shaded region.. DS
question.
27th October repeated
Option 1 : Area of the shaded circular track given as 144 sq.units and difference of 2
radii(why 2) given
Option 2 : Both the radii values given
Answer: both individually,
1: since area is given, side can be foundation and hence the diagonal ( diagonal= radius of
circle passing through the square) and from the given difference in the radio the area of
inner circle can be found)
2: both radii are given, diagonal of the square can be found, area of square and circle can be
found.(repeated on 28th)
(Repeated on 4th)
===========================================================================
=======

NMAT REPEATED QUESTIONS

Q.20 don't remember the question well but secant concept was tested..///
Can someone remember this question?
What was the actual answer? ****************

On 4th Nov, External secant theorem was tested. From a point P, two secant were drawn to
a circle. Was a DS question. Option 1 had 2 lengths, and Option 2 had 1 length. Using both
we can find the 4th length. Lookup Secant theorem, you will understand.

===========================================================================
=======

Q.21 One question to check whether it is a cyclic quadrilateral.(repeated on 28th)


a)AB=AC
b)AM=AN( I missed out N which is opposite to M)
Will EFMN be a cyclic quadrilateral?

Both not sufficient


===========================================================================
=======
Q.22 A is twice as efficient as B, both complete the work in 32 days. How many days b will
take to complete the work alone? ( Repeated 27th oct with changed values) repeated 28th
same values
Ans - 96
(Let number of days taken by A be x and B be 2x
Then 1/2x+1/x=1/32
3/2x=1/32
96=2x
x=48
Hence B will Take 96 days)
Let the efficiency of b be x

NMAT REPEATED QUESTIONS

So for a its 2x
So ratio of time for a:b will b 1:2
Repeated 2day 27 oct
Hence total work is 32x3 =96
And Bs efficiency is 1 so b will take 96 days to complete the work. *+/f
(Repeated on 4th)
===========================================================================
=======
Q.23 Katrina had 8/35 of jessica n jessica's 1/4 of 1/7 is 30000 find katrina's amt.. pretty
simple..must be attempted.
Got repeated today.(27 Oct)
The answer should be 192000? Confirm pls
Yes 192000 is the correct answer.
Repeated on 28th, 2nd Nov
Repeated on 4th Nov.
===========================================================================
=======
Q.24 Three taps P, Q and R can fill tank in X, Y, Z mins they are opened together but Q and
R are closed after 9 mins how long will it take P to fill the tank? Can someone remember
the correct question?
(don't remember the values of XYZ)(approach please) approach -RTW method
Got repeated today. Options were all between 6 - 7 min (6 min 15 sec, 6min 30 sec, 7 min 15
sec and two more). But Q and R were closed after 90 seconds, not 9 mins. Same question
otherwise n for 90 s
And then how much time wo(what will be the answer)???
THIS KIND OF QUESTION REPEATED ON 28TH
Ten find L/x,L/y and L/z (in 1 min ake the lcm of x y and z and let it be L th) for each each
taps.Now for first 9 mins all the three taps are opened so take all the three into account for
first 9 and then L- ( above calc value) will be done only by the P
I had marked 6 m 45s!
Is it right ??
In my q all 3 taps were opened, one of the pipe ( don't remember which ) would take to fill
the remaining amount was asked.
===========================================================================
=======

NMAT REPEATED QUESTIONS

Q.25 What is the max difference between the 4 digit no formed by using
1,2,3,4,5,6,7,8 repetition allowed ( question repeated 27th oct but repetition not allowed)
Ans. 7777 (is this correct)
LARGEST NO. WILL BE 8888 - 1111 = 7777, right ??
The largest 4 digit no can be formed also using the powers right ? (a^b form) NO
===========================================================================
=======

Q.26 (Cannot be greater than 360) a girl travels 360 km in 4 hrs part of the distance by car
and part by train if she had travelled the whole distance by the car then she would have
saved 80% time she spent in the train hence she reached 2 hours early If the velocity of the
car and train are different and constant what is distance that she travelled.
Please explain how saved 80% time = 2 hours? *
Whats the answer? SPEED car= 720km/hr;; train=80km/hr;; is it crt?
Can someone state the question correctly, please? Can anyone remember this question
properly
===========================================================================
=======
Q27. 6 letters, 6 houses. Probability that no house receives correct letter.
this is derangement problem. Answer is 265/720 (this is correct.) yes.
you explain this?
D(n) = (1-1/1! + ! - ! + 1/n! )
D(6) = (1 - 1/1! + ! - ! + . !) = 265
Total ways = 6!
1. necessary if all are positive and none if can be negative ANSWER ???
28. A number is such when, twice the number is divided by 3d it gives a remainder as 20.
And when five times of it is divided by 4d it gives remainder as 32. Find the value of d.
Ans is 12
(i didnt know the method to Probability = 265/6! = 265/720 = 53/144. Plz check no option
was like that in 27th October
===========================================================================
=======

NMAT REPEATED QUESTIONS

Q29.A train crosses platform in 54 sec and man running in opposite direction in 12 sec. if
speed of train is 25kmph and man is 5 kmph what is ratio of length of train to platform?
Answer : Speed of Train - 25* 5/18 m/s ; Ratio - 4:11
Speed of man - 5*5/18 m/s
Let length of train be X
Crossing man running in opp.direction, so
X/[(5*5/18)+(25*5/18)] = 12
X=12*30*5
````````````````````
So x =100 meter
Now
Let the length of platform b Y
So 100+Y/(25*5/18 )= 54
Solve this you will get Y = 275
So X /Y = 100/275 which comes to 4/11
Simple approachLet L-train length & P-platform
54= (L+P)/(25*5/18)....i.e, L+P=375.(i)
Similarly, 12=(L)/(30*5/18)...i.e, L = 100
Putting L=100 in (i) , we get P=275
Required ratio= 100/275...i.e, 4:11(Ans)
Q30.How many 4 digit code can be written if first 2 digit is alphabet and last 2 digit is a
number?
REPEATED ON 28TH
Answer : 26 X 26 X 10 X 10
Ans: 26*26*10*10
Someone said that the question is asked for distinct code
So in that case ans will be 26*25*10*9 it will distinct until and unless repetition allowed is
not given
I dont think we need to multiply by 4! As it is clearly mentioned that first 2 are alphabets
and they are distinct so it will clearly be 26x25; and the last 2 are numbers so they will be
10x9 as they are distinct
26*25*10*9 will be the crect ans for no repeat.
Final answer please? Can someone confirm?
Final ans for dis qtn is 26*26*10*10 for repeatedFinal ans for dis qtn is26*25*10*9 for not repeated

NMAT REPEATED QUESTIONS

===========================================================================
=======
Q31. There are notes of 3 denominations 1C, 50C and 100C..find d number of ways in
which a person can give rs 106 (repeated 27 oct)
Ans : .It should be 4, i think. 4 is the answer.EXPLAIN
1. 1 100C NOTE, 6 1C NOTES
2. 2 50C NOTES, 6 1C NOTES
3. 106 1C NOTES
4. 1 50C NOTE, 56 1C NOTES.
Yes it must b 4 as there are no other option.
===========================================================================
=======
Q32. There is a biased dice..wen d dice is thrown. probability of occurrence of odd numbers
is thrice that of even numbers. If the dice is thrown twice..find the probability of the
products of the numbers appeared being odd.
Answer: 9/16?? 27/64?? Repeated 2nd nov
I think its 9/16 .we need to get all odd numbers , thus the probability will be (3/4)^2
1-x=3x
1-(1/4)=(3/4)
Can someone explain this please?
IS the answer 9/4?
prob(getting odd number)= ; since the product of the numbers should be odd, odd*odd=
odd, that is, * = 9/16
===========================================================================
=======
Q33 x^2 +5x +4 = 0 - find the nature of the roots.(Both are Real and negative and distinct)
Ans: rational unequal
Repeated 2day 27 oct
Approach?
D>0 (D=b^2-4ac for ax^2+bx+c=0)
===========================================================================
=======
Q34 There are 6 ppl P,Q,R,S,T, U..find d no of ways to arrange them such that there are
exactly 2 ppl between R and S. (Repeated on 28th)
Ans.144. Repeated 2nd nov

NMAT REPEATED QUESTIONS

R and S can be arranged in 3*2! = 6 ways


Rest of 4 people can be arranged in 4! Ways
Hence 4!*6 = 24 *6 = 144 ways. Cnfrm (This method is correct.)
Consider this approach
Consider a group of 4 people consisting
of R & S with 2 people between them
_ _ (R _ _ S)
1 2
3
So number of ways of arranging this is 3!
Now, the group of 4 people can be arranged in 4! ways within themselves
(wrong when you take 4! R and S will also change positions e.g. RS_ _. Which is not asked.)
It should be 2(_ _) X 2 (R&S) = 4 and not 4!
Total ways = 3! X 4 = 24 (was this an option?)
So total ways = 3!x4!
= 144
Alternate method:
first choose any 2 other than R & S in 4C2 ways.. these 2 number can arrange between each
other in 2! ways. Now R & S can arrange in 2 ways
R_ _ S or S_ _ R. Now take this 4 letters as one letter so total number of letters 3, which
again can arrange in 3! ways.
Ans is 4C2 * 2! * 2 * 3! = 144
Hope now this is clear to everybody's
Last Approach :p
(R _ _ S)
1

_
2

_
3

consider this as 3 units ..therefore it will arrange itself in 3! ways

Now R and S can arrange themselves in 2 ways R _ _ S or S _ _ R = 2 ways


Now we can pick the people between R and S also.4C2= 6 ways *6 people(R and S cannot
be picked so 4 people) + and we can arrange them in 2! ways .or we can straightaway pick
people between R and S in 4P2 ways=12 ways
What about _ _ should we not take 4C2 for them too? That is coming under the 3! Ways
mentioned in the top.
2 3
So final answer = 3! * 2 * (6 x 2) = 144 ways

NMAT REPEATED QUESTIONS

Dont we need to multiply 2 since r and s too can be interchanged with each other?
Upload.wikimedia.org
===========================================================================
=======
Q35 Mr As (dont remember d name) speed is 2m/s and Mr Bs speed is 4m/s. Both r
walking in d same direction..A train in d same direction passes Mr A in 9s and Mr B in
10s..find the time in which the train passes a pole. ( repeated 27th oct) ( Repeated 28th Oct
) Repeated 2nd nov
Ans. (( I think it should be 90/11) (it is) I also got this. EXPLAIN? Yeah
15sec should be the ans length of train = 120 and speed =8m/s.90/11 must be the
answer,length-180m,speed-22m/s.Can anybody please confirm.
Yeah it is correct
D/x-2 = 9 ; D/x-4 = 10
d= 9x-18
d= 10x-40
Solve this equation and you would get x= 22m/s
Substitute the value of x in any of the equation and you will get length of train (d)= 180
metres
Time taken to cross a pole= 180/22 = 90/11 secs
===========================================================================
=======
Q36 there is a GP series of n terms. Suppose x is the sum of the series. y is the product of
the series and z is the sum of reciprocals of the series. What is y? Options- x/z, (x/z)^n, z/x
something like dat
Ans (x/z)^(n/2)
(x/z)^(n/2)- dis ans was der in the options..can u explain the approach?
Let a = 1st term...GP be of form ar
X = a (r^n-1)/r-1
Y = a^n)*(r^(n*(n-1)/2))
Z = ((r^n -1)/r-1 )* (1/a*r\^(n-1))
Actual answer:[{x/z}^n]^0.5

NMAT REPEATED QUESTIONS

The best method for these type of problems is the substitution method. Yeah for eg
assume series to be 1,2,4 ; n = 3, r = 2 ; a = 1; substitute it in the options...the one which
satisfies is the answer.
===========================================================================
=======
Q37. There are 10 person. If one person with weight 110 left the group and another person
joins the group, the average weight of group decrease by 10. Find the weight of new person
who joined the group.
Answer:10
Let the avg weight of grp b X
Let the weight of person joined the grp b Y
So (10x - 110 + Y)/10 = X -10
.
2110X - 110 + Y = 10 X - 100
Y = 10
===========================================================================
=======
Q38. If a cistern has 3 pipes of 1 cm , 4/3 cm and 2 cm thickness ...where the pipe with 2 cm
thickness can fill the cistern in 2 hrs and 2 min..then in how much time they can fill the tank?
Answer: 1hr 12min? Please confirmyes! 72 min
How, please explain.
Ratio of water flowing through the three pipes respectively
=1^2:(4/3)^2:2^2=1:16/9:4=9:16:36
- - > How do you get this?
(Imagine the opening of the pipe as a square, in one sec, piR^2 volume of water will flow
out)
Suppose
quantity of water flowing through the first pipe in 1 min = 9 litre
quantity of water flowing through the second pipe in 1 min = 16 litre
quantity of water flowing through the third pipe in 1 min = 36 litre
(How can u assume efficiency of all 3 pipe
I think it shud b acc to their ratio ...hence if u are assuming efficiency of 1st pipe as 9 ltr then
accordingly for 2nd pipe it shud b 9* and for 3rd it shud b 18)pls correct if m wrong
biggest tap can fill the tank alone in 122 min.
i.e., capacity of the tank is 122*36 litre
if all pipes are opened together, (9+ 16 + 36) = 61 litre of water will flow in 1 min

NMAT REPEATED QUESTIONS

Therefore, time required = 122*36/61 = 72 min how can you take random values is dre any
hint what values should be taken? Got the same question but 72 was not the question and
put 36 as answer..please confirm.
Hey guys if 2cm thickness can fill the pipe in 122 minutes , added the thickness of all three
pipes it will be around 4.33 which should fill the cistern within 56.35 minutes
Anyone please confirm if it can be right

===========================================================================
=======
Q.39 There are two pipes..A can fill 1/4th of the vessel in 10mins. B can fill 3/4th of the
vessel in 5 mins. If both a n b r left open for 10mins together and then only b is closed. How
much time after this vl the vessel overflow...I marked 1 min as answer n dnt remember
options! (SOMEONE CONFIRM ANSWER?) : This question has inconsistent data, it seems.
Solution:Tap A fills 25% of the vessel in 10 minutes i.e 2.5% per minute
Tap B fills 75% of the vessel in 5 minutes i.e 15% perminute.If this case is considered then
water will start overflowing before the tuning off of tap B.So the question must be B can fill
3/4th of the vessel in 15 minutes.then the solution is feasible and logical.
Tap A fills 25% of the vessel in 10 minutes i.e 2.5% per minute
Tap B fills 75% of the vessel in 15 minutes i.e 5% per minute.
Total Volume being filled 7.5% per minute.
Logically it should be 15 mins for B and in that case ans would be 10 mins. Combined they
fill and remaining A will fill in 10 mins
In 10 Minute when both are working they will fill 75%(3/4th) of the vessel.And to fill the
25%(1/4th) of the vessel A alone would take 10 Minutes as suggested in the question.
HENCE ANSWER WOULD BE 10 MINUTES AFTER B IS CLOSED.

NMAT REPEATED QUESTIONS

Can anyone explain this?

Q. 40.
P&L question was a gold smith adds a mark up of 50% and offers a discount of 10% and also
cheats suppliers and consumers with an improper balance and earns extra profit of 1 gram
per 10 grams from both.
(Solution please!!!!) repeated 27oct
Ans.
I am getting 32% -- its correct
Lets say he has bought 100 gm for 100. CP-100

MP-120 and SP- 108. MARKUP OF 50% is given in the question, why r u taking 20%?Somebody has changed the value it was 20% earlier, hence the solution
He cheats with 1gm for every 10 gm. So while buying he must have gotten 110 gm . and
while selling he sold 90.
Cp for 90gm would be 900/11. And SP is 108 (isme 900 kaise aaya ?? CP per gm would be
10/11)
Profit =32% yes it is correct

Solution FOR 40:- Markup is 50% so MP is 150 where CP is 100 and after 10% discount
SP=135
Now he cheats on both sides so for 100Rs he will get 110g and on selling for 135 he will give
90grams only
So if on 100rs he gets 110g then he will get 90 grams at 900/11Rs which he sells for 135 Rs

NMAT REPEATED QUESTIONS

So profit will be (135-900/11)/900/11 = 1485-900/900 = (585/900)*100% = 65% profit


Please check my answer ! this is correct .bhai phale kisi ne mp 20% mention kiya tha
^^^ haan
mera bhi 65% profit aaya iska ans.yahi sahi hain :) (SAME HERE Mera bhi 65% aaya)
Do look for markup asked, I got 20%, accordingly answer changes :p (28th)
(Repeated on 4th nov, with 20% markup)
****Q41There is a square with side a and one octagon is inscribed in it. What will be
perimeter of octagon.
SOL
Let the side of the octagon be t therefore the side of the square is (a-t)/2 + t + (a-t)/2
The corner makes a 45 degree triangle.
Therefore t = root 2 . (a-t)/2
We get value of t = a/(root2 +1)
Perimeter = 8a/(root2 +1) or 8a(rt 2-1)-- I dont think we will also have to consider - case
also over here? Pls correct me?
-----------------------------------Let me know if any option was 4/3*(1+root2)a
**(not assuming it regular
)**
4 Sides of octagon will be a/3 (the exterior ones)
4 sides of octagon will be root2*a/3(the interior ones)
Summation of all will give perimeter as 4/3*(1+root2)a
Is this correct? Im getting 8a/rt2+1
I am also getting 8a/rt2+1
SolutionLet side of octagon be t
T+ t/rt2 + t/rt2 = a
On solving t= a/(rt2+1)
Therefore perimeter = 8a/(rt2+1)
REPEATED 28TH OCT-What is the correct answer ??
Q.43 In a theatre all rows have same no. Of . One family buys 10 tickets of first two row. 3
members want to sit in first row and 2 in second row. In how many ways they can be seated.
What is the correct answer-144 something was the answer,repeated on 28th
WHICH IS THE CORRECT APPROACH? PLEASE CONFIRM
The question seems incomplete Please suggest!
Is it 2! * 3! * 4! ? (Someone confirm??) (no. of seats in row is not given!)

NMAT REPEATED QUESTIONS

Cant find until no. of seats in a row is given. Plus I think if both rows have 5 seats each .
Then we have to take the selection of seats into consideration as well. So 5c3 * 3! *5c2 *2! *
5!
I think it should be.
Selecting 3 from 10(as they took 10 tickets assuming that there are 10 members) for 1st row
i.e 10c3
And arranging them within 1st row 5 sits
I.e 5p3 and within themselves in 3! Ways
Likewise for next 2 it should b selected from 7c2 ways an arranged in 5P2 ways and within
themselves in 2! Ways.
Please correct me if i am wrong.-How can we assume family members to be 10,may be
there are 5 only?
As they took 10 tickets it indicates that there are 10 members.
Q.44 ab = 832 ?
. A= 26 , b = 32
. a/2 = 13 , b/2 = 16
Above is DS based que..
Either of them is sufficient

CAN SOMEONE PLEASE EXPLAIN THE ABOVE DIAGRAM? WHAT QUESTION WAS IT?

Q.45 x^2 - 5x + 14 = 0, what is the value of x ?


Some option were given for this,Roots will be unreal in this equation.

NMAT REPEATED QUESTIONS

Q46. What is the value for A ?


. A-B < 10
. -A B > 9
Again DS based que.
Sol: Both required?i think its none plz check. (YES ITS NONE)(None because it will have
multiple values and not unique)
Q.47. 1! * 2^2 + 2! 3^2 + 3! * 4^2 + .....+ 20! 21^2 ) / 21 what is the remainder....?
Ans. 19 (is this correct?) yup me too getting it as 19 (APPROACH PLEASE)
19 is correct bro
No. will be divisible by 21 from 6!*7^2 onwards
Simpler Approach :You can write the above equation as
2*2! + 3*3! + 4*4! + . + 21*21!
How !?
\=>

(3! - 2!) + (4! - 3!) + (5! - 4!) + . + (21! - 20!)

=>

- 2! Mod 21 = 19

Even better approach


Expression can be generalised in the following manner
n!*(n+1)2 + (n+1)!* (n+2)2 (n+19)!*(n+20)2
(n+ last no added to n while squaring)
Such questions are based on pattern and patterns can be deduced easily
Take value of n as 1,2,3 form a pattern and see for yourself
Saves a lot of time

Q .48. On a circle 10 equal points are drawn , join 1st with 4th n 2nd with 5th . Intersection
of these cord is named as Q. Find the angle formed by 2nd, Q n 4th
Ans) 144 degree
(Explain this one)
(approach for this?) Each division
is equal to 36 degrees so ans should be 36*2=72 not 144
***answer is 144***.72 is wrong

NMAT REPEATED QUESTIONS

Ans should be 144ans


fj

Explain the diagram please ?


http://mahow-many-triangles-can-be-formed-by-the-vertices-of-a-regular-polygon-of-nsidesth.stackexchange.com/questions/1446438/
CAN SOMEONE EXPLAIN ?
This figure is wrong. Here angle of y will be 72 and not 108 because it covers only 2 sector
and not 3 sector, answer will be 144 final***PERIOD***
144 is confirm as it should be obtuse angle!![simple logic]

Q.49.S=(42+(42+(42+.)^0.5)^0.5)^0.5= x, find x
=> S=(42+S)^0.5
squaring both sides:
S^2=42+S
S^2-S-42=0
solving the quadratic you will get S=7,-6
Ans. 7

NMAT REPEATED QUESTIONS

Q. 50 . Number of triangles that can be formed using vertices of heptagon such that no side
is common to that of the original heptagon. ( repeated 27th oct) ( Repeated 28th Oct )
If one side is common:- number of triangles = (n-4)n

(24th october ka question)

If two sides are common :- number of triangles = n n: no of sides of the polygon


If no side is common(as mentioned in this question)
Number of triangles :n(n-4)(n-5) / 6 is valid for polygon having nn no. of the sides such that n6\
Answer is 7. ( Googled it)
Got repeated today
.
Q51. What day was on Jan 26th 1949
Ans. Wednesday

Q 52. CET- ACA= 281. we were suppose to find the relation between A C T, as in which the
smallest one and which is the largest one. Repeated 27oct
Ans. C>T>A Please provide some explanation.
CET=ACA+281
100C+10E+T=(100A+200) + (10C+80) + (A+1)
Basically, on comparison of 100s place, 10s place & units place, 2+A=C, C+8=E, 1+A=T
Put A=0.. u will get E>C>T>A.
Q. 53 How many words can be made in which there are exactly 2 letters placed Between r
and s. In letters PQRSTU.
Ans 3*2!*4! =144 words ( same as Q.34)
Q.54 Equations of 2 parallel sides of a square were given. ????????????????????????
Area of Square was asked. ( 28th Oct )
8
Q.55 find the 6th term in the series
3,6,12,24
Ans. 96 (Please explain the logic,last no. must be 24 in order to obtain 96 as result).
Is 48 correct ? 3x2,6x2,12x2,24x2
It is a G.P.
48 is wrong answer as he is asking for 6th term not 5th term so 96 is the right answer.
Q- 57 log(base m)X=-log(base(X-1))m.*May be m was in base but structure was same.+
Which of the following options are true.
There were 4 equatio ns.

NMAT REPEATED QUESTIONS

Correct answer was something like: X^2+x-1=0

It was a very easy question, may be I have interchanged the positions of X,m and X-1, but
structure was same-repeated 27oct
Q.58 a clock is set at 6 am on monday.it gains 20 mins in 24 hours.what will be the
actual time on 6 am thursday of same week? ( question repeated on 27th oct but fault
clock lose 20 min was there)
Answer: 5am. Yes it should be 5 Am repeated 27 oct
6. 5.40.
5.20.
5
Mon. Tue.
Wen.
Thursday
Agar Gain hai to5am and lose hai to 7am .right? YES

Q.59 umed invested some amount (given) in a business A.sid joined after few months with
amount (given) end of year profit divided as 5:2(probably). Then they started a
new business B where both invested in 3:5 (probably) and umed withdrew after 8
months.ratio of profit share is 5:8.what is difference between the time sid invested in A and
B? ( repeated 27th oct) ( 28th Repeated ) Repeated 2nd nov
Is the profit invested in B? No the profit is not invested in b. The two are separate scenarios
Whats the correct question ? Anyone ? The que has been corrected.
Q.61 A fills a tank in 10 hours. B fill in 5 hours.today it took 2 hours extra to Fill whole tank
when both were opened together due to a leak.in how many days can the leak empty the
full tank?
8.88 hours ( Got repeated today - 28th Oct -> none of the options were 8.88 or 8.89.
Options were for day77 s. Dont remember the options though Was 0.37 days one of the
option?)

10 & 5 Hours-->LCM is 10. A will do 1 unit of work and B will do 2 unit of work, total 3 units
of work. Time taken when they work together = 10/3 = 3.33 hours. Bcos of leakage , time
taken is 10/3 + 2 hrs = 5.33 hrs.
10/(1+2-X)=5.33 where x is negative work done by leakage.X=1.124
10/1.124 = 8.89 HOURS.(perfect)
,
Q.62 hcf n lcm of two numbers given.one number is given.find the other.
Ans : approach will b

NMAT REPEATED QUESTIONS

Hcf X LCM = X * Y
Q.63 find area of triangle wid co ordinates (0,8) (11,0) (9,0)
Answer:8
X1 = 0 , x2= 11 ,x3= 9
Similarly y1 y2 and y3.
Area of triangle = |x1-x2
y1-y2|
| x2-x3. y2- y3|* is the answer 8 ?
Yes it is
Q.64 9-3log(base3)5- 3^(log(base3)5) (OPTIONS HAVE NUMERIC VALUES )
Are you sure this was the exact question? No numerical value coming with it. Is the answer
0? Someone please confirm. Answer????
Q.65 A can fill half the tank in 10min and B can fill the 1/5th of the tank in 5 min. A and b
are opened for 5 min and then b is turned off. A is open for 15 min more. Find for how much
time the tank overflow?
(I think Qtn is wrng sm wer )
As A can fill half tank in 10 min so full tank can b filled in 20 min
Like wise B can fill it full in 25 min
Let the work to b done is 100 unit(capacity of tank)
So in 1 min A fills 5 ltr and B 4 ltr
In first 5 min tank will b filled (4+5)*5=45 ltr
Remaining is 55 ltr
It can b filled by A in 11min so rest of 4 mins tank will b overflow .
So 1 is wrng ans
Ans- 4 mins is correct
As 1min work = 1/20
Bs 1min work = 1/25
(A+B)s 1min work = 9/100
(A+B)s 5min work = 45/100
Remaining work = 1- 45/50 = 55/100
So this 55/100 work has to be done by A alone, and we need to find in how much time he
did it.
A can do 1/20 of a work in 1min
So 55/100 work will be done in (55/100) * 20 = 11min.
So A will finish off the remaining work in 11 mins.
Overflow time = 15 - 11 = 4min.
Q.66 The average of n consecutive integers are taken by a student but he missed a number
and get the average for (n-1) as 263/13. Find the missing no. ANSWER - 9 How

NMAT REPEATED QUESTIONS

If feel it should be 24.


N-1 = 13 then n=14 let the numbers be x...x+1...x+13 and missing number be y
Hence
their
sum
should
be
14x+91
=
263+Y
Hence x =172+y/14 for x to be integer min value of y= 10 but then x = 13...ie it'll start from
13 so 10 cannot be a missing number...next value of y = 24
I am getting 31..Can anyone pls share the correct approach with answer ??
Anyone with the answer or correct approach?? You get 2 series with this. One when you
take n as 14 and another when you take n as 27.

Q.67 a quadratic equation was given x^2+2x-p=0 (didnt remember the actual coefficient of
x)
whose one of the roots is equal to the positive root of the eq- x^3-3=2x.k Find the other root
of the quadratic eq.ANSWER PLS
Please solve this one(ANSWER)

Q.68 Find a (DS QUESTION)


1)a* (b^25) =0, b is not equal to 0
2)a+b^27+9= b^27+9
Either of statement is sufficient
Q69 a,b,c,d are in Gp. Find (a-b)^2 +(b-c)^2+(d-c)^2-(a-d)^2.Answer - 0
Take a b c d as 1 2 4 8 check with options! (repeated 27th oct)
Q 70 A total sum of 5000 is divided between two person such that total simple interest
received is 1420(didnt remember exactly) then find the individual amount given to
them.what is the ans?
Q71 a triangle formed by coordinates (p,-p)(-1,p^2)(p,-2){actual coordinates were Li'l bit
different} has an area of 12 cm. Find p? ANS? Anyone with the proper question ? Repeat on
28th
Solution!? Solve by determinant rule! Bhai koi example do.. Look above ANSWER ?????
Q71 DS: 6 A B C 24 is a series. Find A, b, c??
(i)B=12
(ii)C is twice of A
I marked this as either one or 2.
The series will be 6, 8, 12, 16, 24(How are they in series?) Repeated on 28th
3rd is a multiple of 1st and 5 th of 3rd

NMAT REPEATED QUESTIONS

------------------------------------------I marked it as both statements together also cannot answer the q. Please confirm.
BOTH ARE REQUIRED: 6,8,12,16,24 (1st term x 2=3rd term, 2nd x 2=4th and so on)
One cant say there will be same pattern in both alternate series
So both not sufficient should be the ans
Wats correct ans ? plz confirm
Ya it will be both not sufficient because the series can be like 6,6rt2,12,12rt2,24(in G.P
common ratio=rt2)
72. A & B throws a pair of dice and winner is one who gets a sum of 9 first.if A starts
throwing first
Then what is the ratio of A winning over B?( repeated 27th oct) repeated 28 th oct morning
slot
Answer- 9:8 < HOW? ANSWER PLEASE??
Yes answer is 9:8 only. Favorable outcome is 4/36 and not favorable is 32/36
So for A,
a=4/3
6
r=1024/1296
= by using formula a/(1-r).
By using this formula we will get probability of both A and B.
For obtaining sum of 9,dice must be rolled out twice.Hence,total outcomes :-36
Now,if B wins,then:A -loses,B-wins:- (34/36)*(2/36)
A loses,B loses,A loses,B wins :- (34/36)*(34/36)*(34/36)*(2/36)
And so on.
Total no. of outcome in which B wins = (34/36)*(2/36)+(34/46)(34/36)(34/36)(2/36)+.........
Applying formula of GP,probability in which B will win:- 17/35.
Thus,probability of A winning over B:- 1-(17/35) = 18/35 (this was not in options. Although i
think your approach is correct) ;;No. Of favourable outcomes will be 4/36 i.e 1/9 . there is
formula for calculating person starting and winning ie. 1 / 2-p where p is probability of
favourable outcomes. Answer will be 9/17 .
Your approach is correct...but the probability is 9/17 not the ratio ...we have to find the
ratio of A winning over B not the probability that A wins...therefore its 9/8

73. Sum of consecutive no.s from 1 to n was given and asked to find out sum of all even no.s
from 1 to n
Ans : n(n+1)/2 = sum given

NMAT REPEATED QUESTIONS

So we will get n
Now N+ N for sum of even nos. (This case N = n/2)
------------------------------------------------------------------------------------------------------------------------------------74). Find the quotient when total no. Of divisors of 16! is divided by right most non-zero
digit of 15!.
Solution pleaseee??
Ans 672. Approach anyone?
Factors of 16!:- 2^15 * 3^6 * 5^3 * 7^2*11^1*13^1
Total no. of factors :- 16*7*4*3*2*2-----> A
Now,Last digit of 15! Will be 8 as 15! Can be written as 2^11*3^6*5^3*7^2*11^1*13*1
Utilizing all 5^3 with 2^3 ,we will be left with 8.
Now divide,A with 8,answer will be 672.
Is 672 the correct answer?? Plz help.
75) 5 ^x + 4^y =odd? Options where for all value even, for all odd value and stuff
76) sum of n numbers was equal to 20306 ( not sure about the value ) then find the sum of
all even number from 1 to n (something like this)
5112 wii be answer, after correcting 20306 (I THINK IT SHOULD BE 20301, THEN N=201)
How 5112 if n=201 so even number will be 100 so sum will be 100* 101 right ?
It shud be 100*101. Anyone else getting the same a0ns????yes it should be 20301.
Yes answer is 100*101=10100 pls explain in detail how 5112 and n=201?
.
77) Allahabad and Varanasi are separated by a 120 km river. It takes a boat 15 hours to do a
complete round trip on one day. But on a certain day, the speed of the stream is twice the
usual. Hence, the time taken by the boat is 24 hours for a complete round trip. Find the
speed of the stream.
a)7.5 b) 8 c) 9 d) 10 e) 7 (all values in m/s)
78) a cylinder and cone combined to form a tank ! Height of tank was 18, height of cone was
6 and they shared the same base with radius 5 , find the total volume
Ans :1100Solution(1320/7??) for this?--> Take height of cylinder
as 12 and rest can be worked out with formula. Answer is 1100 ????????yes 1100
Total capacity will be =volume of cone +volume of cylinder
=(r^2h)/3+r^2h

NMAT REPEATED QUESTIONS

=r^2((6/3)+12)//6 is given and total is 18 so height of cylinder


will be 18-6 i.e 12
=1100

79) a student made a mistake in writing the equation with the coefficient of X and get the
roots 7 and 4 , one more student make a mistake in writing the equation of the constant
and got the answer as 5 and -2 so find the original equation (Repeated on 27th)
Ans: x^2 - 3x + 28 = 0 ??yes its right
The student make mistake in writing coefficient will get product right ans sum wrong and
the one who made mistake in constant will get sum right

80) the fourth number of the ap series is thrice of the first term and the 7th exceeds 2 the
third term by 1 . Find the 9th term.
19/8? Confirm pls. I am also getting 19/8. Me too getting 19/8. Yes it is right answer.
I am getting 19 only..pls give the solution
---correct ans?? Eveln I am getting 19 What is the final answer for this ?
81) few students working together complete and assignment in 60 days 9 more student join
and completed the same assignment in 50 days , working alone how much years will a
student take to complete the assignment (assume 1 year =365) ans:7 years 4 months i guess
1 year =360 would have reduced calculations
Answer : 7.4 years?? (please correct me if Im wrong) sah * Yes,its right 2700 days is the
answer. APPROACH PLS
60x=50(x+9)5 (what is this 5 doing here??, what is the funda going on, can anyone explain)
x=45
total=45*60=2700 days
Ans should be around 9.14 years. What is the final answer for this ?

Q82) A cone was there with height 8 cm and base radius 6 cm. A sphere put inside the cone
such that it touch the sides and get covered fully by the cone. Find the surface
area of the sphere?
Ans : 36 pi ?? EXPLAIN THE APPROACH PLEASE. Answer- 154 cm sq.
Please explain use the logic of similar triangles. From there you will get the radius of the
sphere which is 3.5 cm. Take out TSA from there.- Radius of sphere is coming 3 only
RADIUS will be/ 3.5 as per the formula (hieght +base-hypo)/2= Inradius // How 3.5 is coming
after applying the formula too?

NMAT REPEATED QUESTIONS

Solution please?
Radius comes to be 3 cm exactly. Answer will be 36 pi cm^2.
Can someone please explain in detail?

NMAT REPEATED QUESTIONS

Consider this approach and please let me know if it is correct?


Consider right angled triangle ABF is rotated along the
height AF to give a cone.
Triangle ABF forms a pythagorean triplet
(2 x 3) : (2 x 4) : (2 X 5)
Hence, AB= 10
Now according to the formula
Area = sr
s= semi-perimeter
r= radius of incircle
Area = 1/2 x b x h
= 48
s = (6+8+10)/2
inradius.This

----- We are considering the larger triangle while calculating the


should be (12+10+10)/2=16 and then you get r=3 and hence the ans

36pi.
Please check.
= 12
Hence, r = Area/s

NMAT REPEATED QUESTIONS

= 48/12
=4
So radius comes out to be 4 and it satisfies
the condition DE = EF = 4

Q83 area of a triangle with vertices (0,2),(9,0),(0,13)answer is 49.5 yes right!!! Determinant method ??
Repeated 2day 27 oct
Q84 In an ap, three terms were given and said these 3 are in gp. Find the 21st term of the
ap?
answer-72. Here d came out to be 3 and a =12

Q85 data sufficiency :two spheres are melted to form a combined sphere x. Find the
diameter of x.
St 1)the radius of 2nd sphere was given
St2) the weight of sphere 1 is 8 kg and sphere 2 is 10 kg and they are made of same material
Answer-using both 1 and 2
(i GUESS ONLY STATEMENT ii IS SUFFICIENT AS VOLUME IS GIVEN THE COMBINED VOLUME
WILL BE 18 KG AND THEN WE CAN CALCULATE THE RADIUS TO BE 1.6257)
No bro ...any 1 is correct..in these type of questions we equate the volume
Volume (4/3) pie r^3
Therefore (4/3) pie (r1^3) + (4/3) pie (r2^3) = 4/3 pie (r1+r2)^3 ...so only 1 unknown is
there
Final radius r1+r2 kaise hoga bhai ?
answer????

Q86) The length and breadth of a Rectangle are in the ratio 5 :3. Find the length and breadth
St1) area was given
S2)value of the diagonal was given
Answer-each alone is sufficient

NMAT REPEATED QUESTIONS

Q87)a train crosses one person walking with speed 2 km/hr in 9 sec and another person
with speed 4 km/hr in 10 sec respectively.both persons are moving in same direction as
train.find the time which train crosses a pole. Refer Q35.Repeated on 27th /// IN Q35 the
speed is in m/s ans here is ? Ans ?????
88) a1, a5, a13 (terms of AP) are in GP and a6 + a15 =63. Find a23.
Answer: 182/3. EXPLAIN. a5^2=a1*a13, find d and a using this and above equation you will
get ans 182/3.
89.There is a hexagonal prism of volume 18003 cm and the base is regular hexagon of side
10cm./8*Find height of prism.
Ans was 12.

Angle DAE = 70degree (given)


, area of DEBC = 25
90. If twice of a number divided by 3d gives 20 as the remainder, and five times of the same
number gives 32 as the remainder. Which of the following cannot be the value of d? Options
were like 6,12,15,18. Use trial and error. Ans- None of the above
91. Raj goes from his house to the library at 3 kmph and returns from library to his house at
a speed 33% lesser than his speed during his journey from the house to the librar sey. If the
total time taken for his entire journey is x hours (i dont remember the exact value), what is

NMAT REPEATED QUESTIONS

the distance between his house and the library? Options were like 0.3km, 0.35km, 0.9km,
0.7km
Ans is .3 asked 28th oct how?
X shud be 25mins
92. The highest difference between the 4 digit code formed using the numbers 1,3,4,5,7,8
(no repetition allowed)
Ans: 8754-1345= 7409
93. DS question
Find length of rectangle
1. Diagonal is given
2. Perimeter of rectangle is given
Both not sufficient?
Confirm pls
Both are needed to solve the question. What will be the approach ?
Let diag=10 and 2(l+b)=28 and l^2+b^2=100. Substitute b in terms of l. Solve quad eqn.

94.(1124)^n - (2342)^n where n is non negative integer


Is the answer 3466? Please post the entire question,
95.Two points were given and it is dividing internally with some ratio...check formula
96. There was an INFINITE GP series like
3^ * 9^1/9 * 81 ^ 1/27 * . .. . . . . . . .
Find the product.
Ans : 3 Solution!?

NMAT REPEATED QUESTIONS

Try writing every term in your product as a power of 3. So,


3^(1/3) = 3^(1/3)
9^(1/9) = 3^(2/9)
27^(1/27) = 3^(3/27),
and so forth. Then, your infinite product becomes
3^(1/3) * 3^(2/9) * 3^(3/27) * ...,
which can now be written as
3^[1/3 + 2/9 + 3/27 + ... ]
k = + 2/9 + 3/27.. (1)
Multiply throughout by
k/3 = 1/9 + 2/27 + 3/81 +.... (2)
1. - (2)
2k/3 = + 1/9 + 1/27 + 1/81 +....
2k/3 = / 1 -
2k/3 = /
k=
Final answer = 3^3/4
97. One profit band loss question based on a similar language
Some match boxe have n packets and the person sold some of them at 10 % profit, at what
price he should sell remaining to gain an a certain amount of overall profit. [complete
language i dont remember, but based on this structure+
98. Sum of a series of n terms is 340776 (exact value dont remember, but concept was
same)
Find sum of all even terms till n. I think 583 / 584ANSWER ???
96. A b and c take part in quiz.. what are the marks of A ?
Data sufficiency
1. A^2 + b^2=41

NMAT REPEATED QUESTIONS

C = a + 2 //both solve it and options also i am not able to recall but figures are correct--26th-oct-16 someone please provide solution. IT shud be cannt BD using both as we cant be
sure a is 4 / 5 ?
101) 3 types or rice:
A- 24/kg,B-30/kg,C-x/kg in ratio 1:1:3
Resultant -36/kg
Total weight 120kg
find x?
For Such Questions, always use the Options.
S1: Avg Value per Kg > A,B. So Cs value per Kg >A,B.
Look at options. Here we will consider 42 directly.
Ratio of All three weights is 1:1:3. So total weight 120 is divisible by 5.
Weight of C in the mixture is (120) = 72 Kg. A:B = 1:1, so they are 24 Kgs each.
Now, use weighted average concept.
We have assumed C as 42/Kg.
[(42 * 72) + (24*24) + (24 * 30)]/(120)
=> 4320/120 = 36 Kgs.
Hence proof of answer clarifies 42 as the Ans.
Answer: 42.
1.
102) male n female ratio in an office 5:3
If same number of males and females are added in the office.. What shud be ratio
now - I got answer 3:2
d the (WONT C) IT SHUD BE CANNT BE
DETERMINEDplease confirm
103) distance between A-b -- 120 km
Boat speed - 6kmph On 1 day, Round trip done in 15 hours
Next day speed of steam is twice - round trip - 24 hours
What is speed of stream.? (Question doesnt seem to be right, stream wont effect the
time taken for round trip) please confirm - it will affect and boat speed was not given
104) Round trip from a to b in 3 hours
A-b 24km
Speed of boat? 8 k/h
105) Max diff between 4 digit number formed using 1,3,4,5,7,8 ( no Repetition
allowed)
Ans: 7409

NMAT REPEATED QUESTIONS

106) 6 boys n 10 girls complete job in 12 days. They completed 50% of job
They are joined by 2 more boys n girls each and now 2/3rd of the remaining work is
completed Remaining work is done by 1girl in how many days? ANSWER ???
8
Similar question as above, but for the remaining work how many more girls are
needed? (Nov 4)
107) ax^2+bx+c =0
Roots are p & q
Find the equation whose roots are P^4 & q^4...

Options were very lengthy so I'm not able to recollect


108) x^2 + bx + c =0
One guy calculated and got the roots as 7 and -4..
But he took the wrong value of 'c'
Other guy got roots as 7 and 6..
But he took the wrong value of 'b'
Find the correct equation
REPEATED ON 29TH OCT)

I got x^2 -3x +42 as the answer


its correct
109-Train crosses two men in the Same direction as John with 2kmp and Sam with 3kmp in
9s and 10s resp. Find the speed of train crossing a pole? Speed of train=12, and time to cross
the pole=7.5 sec. Please confirm.
110-Gaya completes a work I the same time as done by both Dekha and Shruti. Dekha is
2time as effectient as Shruti. Then which of the following is true:
A.
GAYA IS THREE TIMES AS EFFICIENT AS SHruti?
B.
GAYA IS twice efficient than Shruti Ans.Gaya three times as efficient as shruti. Please
confirm.
111- A (don't remember name) brought some number of mugs from dealer H at 1200rs .
Out of which 10 mugs had cracks . Remaining were sold at 2rs more than the cost price
making an overall profit of 60rs . Again A brought same no of mugs at rs 10 more than he
brought earlier from another dealer. No mugs had crack this time. So at what price shall he
sell each mug so as he can earn an overall profit of 20% in transaction?

NMAT REPEATED QUESTIONS

Todays Q(2nd Nov) ans?


Quant
Some plant bore fruits after 32 days and flowers after 44 days, if the first time it bore fruit
and flower together it was Wednesday, when would it happen next (Nov 4)
Out of 130 people, even number were below 20, multiples of 5 were something 1, multiples
of 7 were something 2. Number of people ABOVE 20, and NOT something 1 and NOT
something 2 (Nov 4)
Q.
A guy got 25% of the maximum marks, and failed by 60 marks. Another guy got 42% of the
total marks, and passed by 8 marks. What were the maximum marks of the test?
DI (Nov 4)
Set 1. 30% reduction
Set 2. Utilised buses something
Questions
1. Difference between state A Y1 and Y2 un-utilised buses
(1450-800) = 650
2. Then something like 15% of the un-utilised buses were overaged buses, so find utilised
overaged buses, 3_17
Set 3
Two piecharts were given and increase decrease percentage (comparatively easier)
1. Which of the above measures has a ratio 5:4, so it was one with 15% and 12%
Set 4
Some latin america and europe exports question
Set 5 rainfall(mm)/hr, total rainfall(mm) and rainy days, for 5 months, for calculating hrs of
rainfall it was something like total rainfall/rainfall per hr

NMAT REPEATED QUESTIONS

NMAT LOGICAL REASONING WINDOW- 4


1.All the questions today were lengthy.
2.Lengthy Verbal reasoning-Lot of Inferences and weakening and strengthening
arguments(Less of concludable facts,implicit assumptions and course of actions.
3.1 puzzle was easy
4.Time consuming singlets ...for example one matrix was there we had to identify the
number of letters from the matrix but after transposing the matrix...also this matrix had no
numbering for rows and columns...we had to give the numbers...so in short the whole of LR
was time consuming

NMAT LOGICAL REASONING WINDOW- 2


Logical Reasoning
Can someone please brief about the LR structure.
The below question was there .. nos were 7, 19, 22 output, step3, step 4 was asked
.
1. Input Output question :
Input

45

Step 1

09

Step 2-->Logic for this step please?

20(4*5)

Step 3

29

NMAT REPEATED QUESTIONS

Step 4

38

Similar logic was present in Paper with 5*5 matrix


2. Decoding - encoding question was of logic where alphabets were shifted by n+1, n+2,
n+3,
3. Raju and Ravi decided to meet at coffee shop. Ravi travelled some directions and reached
shop. Raju travels some direction from his house. What is direction of Raju relative to Ravi ?
Ans. South east (approach?)
4. Five kids with some names play five sports. Every sport is indicated by a geometry shape.
Relation needs to be established among kids, sports and shapes. Repeated question on 23
oct
CAN SOMEONE POST THE RELATIONSHIP OF KID WITH THE RESPECTIVE SHAPE AND SPORT.
5. 10 teams are divided into two groups. Baseball is played among them such that two
teams from each group qualifies for quarter finals. Games are played at different cities of
India. Relation needs to be established between position of teams, cities at which games
were played.repeated question 23 oct...
6. There is one couple in this 3 generation of family. SAP is grandfather of JAVA and father of
ORACLE. Sidbell was grandmother of JAVA. (There was one more clause) . question was to
identify the statement which cannot be true.one option said the grandmother( cant
remember name) is a male.thats the answer
.
7. Some symbols were given , triangle , circle , square and rectangle. 4 kids- Ryan , john,
geeshu,sandy. Ages - 6,7,13,15 . 4 games - roller coaster, giant wheel,splash, crazy car. Some
conditions were given acc to that have to arrange these people..overall easy set
8 . Input output question
Input - 13
Step 1 - 65. ( 13*5)
Step 2 - 3 . ( Multiply non zero digits)
Step 3- 7 ( add digits of step 1) should be 11
Step 4 - 14 add step 1 and 2
Step 5 - 21 again addition of step 1 and 3
steps..
like this input output repeated on 27th
For above..I don't remember the question exactly..but it was on similar logic.. sequence of
steps might be different .but overall easy..
9. 2 questions on blood relations with symbols showing the type of relation.
=>>Lady is daughter of the wife of my grandfatherss only son How is she related to me?
Can anyone please provide the Q. ?

NMAT REPEATED QUESTIONS

MOTHER (It must be sister. Plz confirm?)

10. Critical reasoning questions were moderate. One was about course of action for union
minister and about concentration on music while driving
11. Many coding and decoding questions were there one of them was based on +1, -2, +1,
-2
Another was based on +15, -1, +5, another was on +2,-2,+2,-2.
For these questions write down A -1 ,B-2,C-3,D-4,E-5,F-6 .. Y-25 , Z-26, then is very easy to
crack.
12. Many verbal reasoning questions were there assumptions, which statement is shows
strength, weaken the argument
13. a certain object was passed between 5 places, namely, new york, paris, istanbul, london
and cairo and 5 modes of transport were given with their cost. we were supposed to pair
the places with their respective cost and mode of transport.
14. there were 5 mothers having 2 child each and their ages were given. again we had to
find their respective ages and a proper pairing between mother and the child.
15. I/O Question
GO is the input
Step 1- sum of digits =7+15 i.e, 22
Step 2 - 2 can be represented as b.
So bb is the next step
Step 3 - And then input is combined with bb, so gobb
Step 4 - Represent it with respective numbers (71522)
Step 5 - Then sum of those digits in the next step (17) ---> Q
And the represent the sum with the alphabet which represents that number , ex- if the sum
is 13, represent it with m
154. 110,152,204 ,?explain
Ans 246 .. how?

16. Raju has a birthday party.but all the guests are late.the only son of his grandfather gave
wrong address to cab driver.similarly some lady was late in office.and relation een the lady
and only son is asked.
the lady was probably rajus motherA.
Actyually it should be Cannot be determined . PLZ Confirm.

NMAT REPEATED QUESTIONS

17. One coding question which was something as mars is satellite is 743 and one other
statement.code for satellite was asked which was 719.
18.input/output question (same logic follows though i have taken random input)....this word
fico 45 years will be ur first ques(just to recognise this i/o set) ( repeated 27th oct) 28th
repeated
input

Fico 45 years

Age 30 years

step-1

445

330

step-2

80

step-3

step-4

640

81

step-5

568

81

Solution
Step-1 (no of letter in the input word)*100+...... years
Step-2 multiply all non zero digits of step-1
Step-3 sum of digits in step-2
Step-4 step2*step3
Step 5 step4-(step2-step3)
Step 5 is wrong here. Can anyone arrive at the logic correctly? Its correct
Same type of question was asked on 28th with ILTS
19. 62+8,70+7,?,91+24
Ans. QA
20. A,B,C,D,E,F are sitting across a round table facing center O. A and D are immediate
neighbours of E. A is sitting to the right of E. G in not between F and C. Which of the
following pair has the second person sitting to the immediate right of the first person?
Ans. EA

n the LR section, two puzzle sets had 3 variables each. One with five companies, five colors
and five other things. We had to link all and get the structure. Other puzzle was also on the
same concept. Each will take atleast 4-5 minutes to crack. Lost on a lot of time because of
this and cracked only one correctly. So watch out.
The DI part was VERY EASY. One set had 2 pie charts. Revenue for 2005 (one pie chart) was
10L and Revenue for 2006 was 12.5L. The pie charts had retail, programming, manufacturing

NMAT REPEATED QUESTIONS

and 3 other details (cant remember) for each in different percentage. Very simple
percentage based questions were asked like find increase in revenue for programming, or
which one had the same revenue in 2005 and 2006 (this was programming mostly, the
answer came to 1,50,000 for both years) but you can easily calculate. Then one other
question was 25% revenue in 2005 and 2006 was given to ocharity by manufacturing, find
difference in their revenues then.
The other DI set was a bar graph of 3 companies ABC. I don't recall for which commodity,
but main thing was 30% of the purchase by any of these was disposed after 3 years. Then
questions were asked based on that.
21. A lot of questions similar to this:
So, four numbers were given - A,B,C,D and you have to find their relation and find out the
value for ?
Also, in one of the question, three similar figures were given and their was a ? in the third
image. So, again, find the relation and value for ?.
22. There was a question where it was given that X and Y go to Beijing; A and X go to
HongKong (I dont remember the places so I am writing X, Y and A). In that there were two
figures respectively with three overlapping figures. In case of Beijing the middle figure
(which is being overlapped by the other two figures) has 10 sides. Beijing has 7 letters and
Hong Kong has 8 letters and the middle figure had 12 sides.
23
Input: Two numbers were given
S1: Take the difference between the 2.
S2: Sum the digits in S2
S3: The difference of S3 and S1 and adding it to S2
S4: Sum of digits of S3 (it was something like this. I dont remember exactly but it was of this
pattern
I/O questions (Nov 4)
No1, No2
Step 1 difference between the two
Step 2 max(no1,no2) + step 1
Step 3 add digits of step 2
Step 4 add step 3 and step 1
Step 5 min(no1,no2) + step 4
Some company with A,B,C,D,E units - marketing,finance, retail,life science, infrastructure
with some labels - gold, strategic, platinum, new (Nov 4)
A life science - gold
B retail - strategic
C finance - platinum

NMAT REPEATED QUESTIONS

D marketing
E infrastructure - new
A, B,C,D,E -font size and color (Nov 4)
A - gothic or calibri red
B - gothic or arial blue
C - universal yellow
Dgreen
E - Times New Roman purple
Questions :
1. Color of D
2. Correct combination - C
3. For how many the font cannot be found - 3(A,B,D)

Q.
That productivity of a factory is decreasing because there's this new game on smartphones,
and workers are busy playing what should the authorities do. Ban smartphones/counsell
them/ban the network/ban the game etc etc.

NMAT LOGICAL Data Interpretation-WINDOW 2

The DI part was VERY EASY. One set had 2 pie charts. Revenue for 2005 (one pie chart) was
10L and Revenue for 2006 was 12.5L. The pie charts had retail, programming, manufacturing
and 3 other details (cant remember) for each in different percentage. Very simple
percentage based questions were asked like find increase in revenue for programming, or
which one had the same revenue in 2005 and 2006 (this was programming mostly, the
answer came to 1,50,000 for both years) but you can easily calculate. Then one other
question was 25% revenue in 2005 and 2006 was given to ocharity by manufacturing, find
difference in their revenues then.
The other DI set was a bar graph of 3 companies ABC. I don't recall for which commodity,
but main thing was 30% of the purchase by any of these was disposed after 3 years. Then
questions were asked based on that.
21. A lot of questions similar to this:
So, four numbers were given - A,B,C,D and you have to find their relation and find out the
value for ?
Also, in one of the question, three similar figures were given and their was a ? in the third
image. So, again, find the relation and value for ?.

NMAT REPEATED QUESTIONS

22. There was a question where it was given that X and Y go to Beijing; A and X go to
HongKong (I dont remember the places so I am writing X, Y and A). In that there were two
figures respectively with three overlapping figures. In case of Beijing the middle figure
(which is being overlapped by the other two figures) has 10 sides. Beijing has 7 letters and
Hong Kong has 8 letters and the middle figure had 12 sides.
23
Input: Two numbers were given
S1: Take the difference between the 2.
S2: Sum the digits in S2
S3: The difference of S3 and S1 and adding it to S2
S4: Sum of digits of S3 (it was something like this. I dont remember exactly but it was of this
pattern
I/O questions (Nov 4)
No1, No2
Step 1 difference between the two
Step 2 max(no1,no2) + step 1
Step 3 add digits of step 2
Step 4 add step 3 and step 1
Step 5 min(no1,no2) + step 4
Some company with A,B,C,D,E units - marketing,finance, retail,life science, infrastructure
with some labels - gold, strategic, platinum, new (Nov 4)
A life science - gold
B retail - strategic
C finance - platinum
D marketing
E infrastructure - new
A, B,C,D,E -font size and color (Nov 4)
A - gothic or calibri red
B - gothic or arial blue
C - universal yellow
Dgreen
E - Times New Roman purple
Questions :
1. Color of D
2. Correct combination - C
3. For how many the font cannot be found - 3(A,B,D)

NMAT REPEATED QUESTIONS

Q.
That productivity of a factory is decreasing because there's this new game on smartphones,
and workers are busy playing what should the authorities do. Ban smartphones/counsell
them/ban the network/ban the game etc etc.

Vous aimerez peut-être aussi